Annotation of db/baza/sibst15.txt, revision 1.4

1.1       rubashki    1: Чемпионат:
                      2: Студенческая лига Сибири - 2015/16
                      3: 
                      4: Дата:
                      5: 09-Oct-2015 - 25-Mar-2016
                      6: 
                      7: Тур:
                      8: 1 тур
                      9: 
                     10: Дата:
                     11: 09-Oct-2015
                     12: 
                     13: Редактор:
                     14: Мишель Матвеев (Санкт-Петербург)
                     15: 
                     16: Инфо:
                     17: Редактор благодарит за тестирование и полезные замечания Антона
                     18: Тахтарова, Александра Мудрого, Александра Камаева, Ирину Зубкову, Юлию
                     19: Лободу, Андрея Данченко, Илону Косенко, Сергея Лобачёва, Садига
                     20: Гамидова, Анастасию Мартынову, Анастасию Белозёрову, Дмитрия Арбузова,
                     21: Дмитрия Фёдорова, Евгения Мясоедова, Виталия Эйбера.
                     22: 
                     23: Вопрос 1:
                     24: Ритуальное приветствие, с которым простой фиджиец обращался к вождю,
                     25: совпадает с фразой, которую однажды прочитала ОНА. Назовите ЕЕ.
                     26: 
                     27: Ответ:
                     28: Алиса.
                     29: 
                     30: Комментарий:
                     31: Приветствие - "Съешь меня!". Среди аборигенов был широко распространен
                     32: каннибализм.
                     33: 
                     34: Источник:
                     35: http://en.wikipedia.org/wiki/Fiji
                     36: 
                     37: Автор:
                     38: Мишель Матвеев (Санкт-Петербург)
                     39: 
                     40: Вопрос 2:
                     41: В одном описании начала боксерского поединка на лице боксера появляется
                     42: ОНА. ОНА фигурирует в названии фильма, главного героя которого играет
                     43: актер, также связанный с боксом. Назовите ЕЕ двумя словами.
                     44: 
                     45: Ответ:
                     46: Первая кровь.
                     47: 
                     48: Комментарий:
                     49: Главную роль в фильме "Рэмбо: Первая кровь" играет Сильвестр Сталлоне,
                     50: который также сыграл боксера Рокки.
                     51: 
                     52: Источник:
                     53:    1. http://www.sport-express.ru/newspaper/2009-04-06/16_1/
                     54:    2. http://ru.wikipedia.org/wiki/Сталлоне,_Сильвестр
                     55: 
                     56: Автор:
                     57: Мишель Матвеев (Санкт-Петербург)
                     58: 
                     59: Вопрос 3:
                     60: Некий физик, прилетев в родной город, сел на заднее сиденье своей
                     61: собственной машины и какое-то время ждал шофера. Из какого города он
                     62: вернулся?
                     63: 
                     64: Ответ:
                     65: Из Стокгольма.
                     66: 
                     67: Комментарий:
                     68: Там он получил Нобелевскую премию. Лауреатам выделяют, в частности,
                     69: личные лимузины с шоферами. К хорошему быстро привыкаешь...
                     70: 
                     71: Источник:
                     72: http://www.gazeta.ru/science/2011/12/12_a_3924654.shtml
                     73: 
                     74: Автор:
                     75: Мишель Матвеев (Санкт-Петербург)
                     76: 
                     77: Вопрос 4:
                     78: Один рекламный слоган гласит: "Мой муж пьет, но я за него спокойна!".
                     79: Какой товар рекламируется при помощи этого слогана?
                     80: 
                     81: Ответ:
                     82: Фильтр для воды.
                     83: 
                     84: Зачет:
                     85: "Аквафор".
                     86: 
                     87: Комментарий:
                     88: Муж пьет воду, но не абы какую, а безопасную для здоровья.
                     89: 
                     90: Источник:
                     91: Реклама в метро Санкт-Петербурга.
                     92: 
                     93: Автор:
                     94: Мишель Матвеев (Санкт-Петербург)
                     95: 
                     96: Вопрос 5:
                     97: Платон утверждал, что самомнение - спутник ЭТОГО. Назовите Нобелевского
                     98: лауреата, написавшего роман, в названии которого фигурирует ЭТО.
                     99: 
                    100: Ответ:
                    101: [Габриэль] Гарсиа Маркес.
                    102: 
                    103: Комментарий:
                    104: ЭТО - одиночество.
                    105: 
                    106: Источник:
                    107:    1. М.Л. Гаспаров. Записи и выписки.
                    108: http://www.flibusta.net/b/244208/read
                    109:    2. http://ru.wikipedia.org/wiki/Гарсиа_Маркес,_Габриэль
                    110: 
                    111: Автор:
                    112: Мишель Матвеев (Санкт-Петербург)
                    113: 
                    114: Вопрос 6:
                    115: Согласно песне Ольги Арефьевой, он всегда попадает в цель и никогда не
                    116: садится на мель. Последнее, очевидно, в переносном смысле. Назовите его.
                    117: 
                    118: Ответ:
                    119: [Вильгельм] Телль.
                    120: 
                    121: Комментарий:
                    122: Фамилия "Телль" в этой песне рифмуется со словами "цель" и "мель".
                    123: Телль, как известно, был прекрасным стрелком.
                    124: 
                    125: Источник:
                    126: http://www.ark.ru/ins/lyrics/VilgelmTell.html
                    127: 
                    128: Автор:
                    129: Мишель Матвеев (Санкт-Петербург)
                    130: 
                    131: Вопрос 7:
                    132: Когда Диего Марадону впервые просматривали в профессиональную футбольную
                    133: команду, тренер решил, что это обман и Марадона является ИМ. В
                    134: действительности Марадона не является ИМ, хотя ближе к этому, чем
1.4     ! rubashki  135: среднестатистический человек. Кем - ИМ?
1.1       rubashki  136: 
                    137: Ответ:
                    138: Карликом.
                    139: 
                    140: Комментарий:
                    141: Марадона играл так хорошо, что тренер решил, что перед ним не подросток,
                    142: а взрослый игрок, являющийся карликом. Рост Марадоны - 167 см.
                    143: 
                    144: Источник:
                    145:    1. http://ru.wikipedia.org/wiki/Марадона,_Диего_Армандо
                    146:    2. http://en.wikipedia.org/wiki/Human_height
                    147: 
                    148: Автор:
                    149: Мишель Матвеев (Санкт-Петербург)
                    150: 
                    151: Вопрос 8:
                    152: Мотогонщик Спис стал победителем сезона в трековых гонках. За свой успех
                    153: он получил, в частности, драгоценный приз и прозвище. Такое же прозвище
                    154: получил, например, Коби Брайант. Какое?
                    155: 
                    156: Ответ:
                    157: Властелин колец.
                    158: 
                    159: Зачет:
                    160: Властелин кольца.
                    161: 
                    162: Комментарий:
                    163: Соответственно, трековых и баскетбольных. Спису подарили золотое кольцо
                    164: с монограммой - неплохой аналог Кольца Всевластья.
                    165: 
                    166: Источник:
                    167:    1. http://5mv.ru/article.php/Nichya_v_kotoroy_ne_doljno_bylo_ostat
                    168:    2. http://www.basketball.ru/articles/reprint/reprint_3892.html
                    169: 
                    170: Автор:
                    171: Мишель Матвеев (Санкт-Петербург)
                    172: 
                    173: Вопрос 9:
                    174: Согласно шутке, когда девушке предложили исполнить любое ее желание с
                    175: условием, что соседка получит в два раза больше, эта девушка попросила
                    176: при ближайшем случае одну... Что?
                    177: 
                    178: Ответ:
                    179: Полоску.
                    180: 
                    181: Комментарий:
                    182: В тесте на беременность. Одна полоска означает отсутствие беременности,
                    183: две - наличие.
                    184: 
                    185: Источник:
                    186: http://www.anekdot.ru/id/481470/
                    187: 
                    188: Автор:
                    189: Мишель Матвеев (Санкт-Петербург)
                    190: 
                    191: Вопрос 10:
                    192: По мнению СенЕки, Катон Младший ДЕЛАЛ ЭТО с государством. Сенека мог бы
                    193: сравнить Катона с мифологическим персонажем, в обязанности которого
                    194: входило ДЕЛАТЬ ЭТО... С чем?
                    195: 
                    196: Ответ:
                    197: С небом.
                    198: 
                    199: Комментарий:
                    200: По мнению философа Сенеки, Катон Младший, образцовый римский гражданин,
                    201: своим примером держал государство на своих плечах. Атлант на своих
                    202: плечах держал небо.
                    203: 
                    204: Источник:
                    205: К.В. Душенко. Мысли, афоризмы и шутки знаменитых мужчин.
                    206: http://www.flibusta.net/b/120908/read
                    207: 
                    208: Автор:
                    209: Мишель Матвеев (Санкт-Петербург)
                    210: 
                    211: Вопрос 11:
                    212: Согласно Евгению Клюеву, страна ПерУ страдала от недостатков в
                    213: планировке зданий, однако потом произошла ОНА. Назовите ЕЕ придуманным
                    214: Клюевым неологизмом.
                    215: 
                    216: Ответ:
                    217: Перустройка.
                    218: 
                    219: Комментарий:
                    220: Перестройка в Перу.
                    221: 
                    222: Источник:
                    223: Е.В. Клюев. Странноведение.
                    224: 
                    225: Автор:
                    226: Мишель Матвеев (Санкт-Петербург)
                    227: 
                    228: Вопрос 12:
                    229: Цитата. "Однажды в апреле две тысячи двадцать второго тяжелая дверь
                    230: библиотеки оглушительно хлопнула". Следующая фраза почти совпадает с
                    231: названием знаменитого рассказа того же автора. Напишите это название.
                    232: 
                    233: Ответ:
                    234: "И грянул гром".
                    235: 
                    236: Комментарий:
                    237: Знаменитый рассказ Рэя Брэдбери. Тематикой книг и будущего Брэдбери
                    238: известен, в частности, благодаря роману "451 градус по Фаренгейту", хотя
                    239: процитирован не он.
                    240: 
                    241: Источник:
                    242:    1. Р. Брэдбери. Лучезарный Феникс.
                    243: http://www.flibusta.net/b/8786/read
                    244:    2. http://en.wikipedia.org/wiki/A_Sound_of_Thunder
                    245: 
                    246: Автор:
                    247: Мишель Матвеев (Санкт-Петербург)
                    248: 
                    249: Вопрос 13:
                    250: Говоря об особенностях мышления ученого, Станислав Смирнов упомянул
                    251: Пуанкаре, впрыгивающего в омнибус, а также человека, совершившего, можно
                    252: сказать, противоположное действие. Кого именно?
                    253: 
                    254: Ответ:
                    255: Архимеда.
                    256: 
                    257: Комментарий:
                    258: Выпрыгнувшего из ванной. Смирнов объяснял, что ученый часто размышляет
                    259: над проблемой попутно с какими-либо бытовыми занятиями. Пуанкаре решил
                    260: достаточно сложную задачу, впрыгивая в омнибус.
                    261: 
                    262: Источник:
                    263: "Санкт-Петербургские ведомости", 16.09.2011 г.
                    264: 
                    265: Автор:
                    266: Мишель Матвеев (Санкт-Петербург)
                    267: 
                    268: Вопрос 14:
                    269: Одна женщина, персонаж Эмиля Кроткого, была образцом постоянства. Тем не
                    270: менее, к ней была применима известная поговорка, потому что у этой
                    271: женщины кое-что имелось лишь в одном экземпляре. Что именно?
                    272: 
                    273: Ответ:
                    274: Перчатки.
                    275: 
                    276: Зачет:
                    277: Перчатка.
                    278: 
                    279: Комментарий:
                    280: Меняла любовников, как перчатки, но перчатки у нее были одни.
                    281: 
                    282: Источник:
                    283: http://www.aphorism.ru/comments/mj541kk851.html
                    284: 
                    285: Автор:
                    286: Мишель Матвеев (Санкт-Петербург)
                    287: 
                    288: Вопрос 15:
                    289: Когда Гней Помпей справлял африканский триумф, он сначала хотел
                    290: использовать ПЕРВЫХ, но затем пришлось поменять их на ВТОРЫХ. Впрочем,
                    291: считается, что обмен ПЕРВОГО на ВТОРОГО - примерно равноценный. Назовите
                    292: ПЕРВЫХ и ВТОРЫХ в правильном порядке.
                    293: 
                    294: Ответ:
                    295: Слоны, кони.
                    296: 
                    297: Комментарий:
                    298: Для четверки слонов городские ворота оказались слишком узки, поэтому
                    299: пришлось въезжать традиционным образом - на четверке коней. В шахматах
                    300: конь и слон считаются фигурами примерно равной силы.
                    301: 
                    302: Источник:
                    303:    1. Плутарх. Сравнительные жизнеописания. Помпей.
                    304: http://ancientrome.ru/antlitr/t.htm?a=1439003200
                    305:    2. http://ru.wikipedia.org/wiki/Шахматные_фигуры
                    306: 
                    307: Автор:
                    308: Мишель Матвеев (Санкт-Петербург)
                    309: 
                    310: Вопрос 16:
                    311: Герой Вудхауза говорит: "Вы следуете за ходом моих рассуждений?". В
                    312: следующей своей фразе он, как считается, впервые употребляет знаменитое
                    313: выражение. Напишите это выражение, в котором фигурирует имя собственное.
                    314: 
                    315: Ответ:
                    316: "Элементарно, Ватсон".
                    317: 
                    318: Зачет:
                    319: "Элементарно, Уотсон".
                    320: 
                    321: Комментарий:
                    322: Главный герой ведет рассуждения, подобно Шерлоку Холмсу, и имитирует его
                    323: манеру общения с Ватсоном. При этом Вудхауз придумал настолько удачную
                    324: фразу, что она впоследствии стала неразрывно связана с образом Холмса,
                    325: хотя сам Конан Дойл ее не использовал.
                    326: 
                    327: Источник:
                    328:    1. П.Г. Вудхауз. Псмит-журналист.
                    329: http://www.flibusta.net/b/119155/read
                    330:    2. http://ru.wikipedia.org/wiki/Вудхаус,_Пелам_Гренвилл
                    331: 
                    332: Автор:
                    333: Мишель Матвеев (Санкт-Петербург)
                    334: 
                    335: Вопрос 17:
                    336: Сергей Лукьяненко приводит мнение, что основных сюжетов в мировой
                    337: литературе немного, и одним из них является осада крепости. В качестве
                    338: примера он приводит произведение, сюжет которого - поочередная осада
                    339: трех крепостей. Назовите это произведение.
                    340: 
                    341: Ответ:
                    342: "Три поросенка".
                    343: 
                    344: Источник:
                    345: С.В. Лукьяненко. Черновик.
                    346: http://www.loveread.ec/read_book.php?id=37674&p=80
                    347: 
                    348: Автор:
                    349: Мишель Матвеев (Санкт-Петербург)
                    350: 
                    351: Вопрос 18:
                    352: Энциклопедия объясняет, что ОНИ попали в Европу через Кордовский
                    353: халифат. Назовите ИХ двумя словами.
                    354: 
                    355: Ответ:
                    356: Арабские цифры.
                    357: 
                    358: Комментарий:
                    359: В действительности они происходят из Индии, но в Европу пришли через
                    360: арабов, за что и получили свое название.
                    361: 
                    362: Источник:
                    363: БЭКМ-2003, ст. "Цифры".
                    364: 
                    365: Автор:
                    366: Мишель Матвеев (Санкт-Петербург)
                    367: 
                    368: Вопрос 19:
                    369: В одном романе об Эрасте Петровиче Фандорине фигурирует грузинская
                    370: княжна. Назовите ее фамилию.
                    371: 
                    372: Ответ:
                    373: Чхартишвили.
                    374: 
                    375: Комментарий:
                    376: Совпадает с настоящей фамилией автора романа.
                    377: 
                    378: Источник:
                    379: Б. Акунин. Особые поручения. http://www.flibusta.net/b/187523/read
                    380: 
                    381: Автор:
                    382: Мишель Матвеев (Санкт-Петербург)
                    383: 
                    384: Вопрос 20:
                    385: Считается, что для корректного измерения артериального давления
                    386: необходимо часовое воздержание от НИХ. "ОНИ" - сборник из одиннадцати
                    387: бесед. Назовите ИХ.
                    388: 
                    389: Ответ:
                    390: Кофе и сигареты.
                    391: 
                    392: Комментарий:
                    393: Кофеин и никотин повышают артериальное давление. Персонажи фильма "Кофе
                    394: и сигареты" ведут беседы за кофе, куря сигареты.
                    395: 
                    396: Источник:
                    397:    1. http://www.tensoval.ru/how_to_measure.php
                    398:    2. http://ru.wikipedia.org/wiki/Кофе_и_сигареты_(фильм)
                    399: 
                    400: Автор:
                    401: Мишель Матвеев (Санкт-Петербург)
                    402: 
                    403: Вопрос 21:
                    404: Шутка с российского сайта: "Мой дедушка во время Великой Отечественной
                    405: войны подорвал вражеский велосипед и был награжден...". Чем?
                    406: 
                    407: Ответ:
                    408: Педалью.
                    409: 
                    410: Источник:
                    411: http://www.anekdot.ru/id/93257/
                    412: 
                    413: Автор:
                    414: Мишель Матвеев (Санкт-Петербург)
                    415: 
                    416: Вопрос 22:
                    417: В одном заговоре участвовали многие знатные вельможи и даже сама
                    418: королева. Одним из результатов раскрытия этого заговора стало появление
                    419: одетых в красное. Кого именно?
                    420: 
                    421: Ответ:
                    422: Гвардейцев кардинала.
                    423: 
                    424: Комментарий:
                    425: После раскрытия заговора против Ришелье была создана его личная охрана,
                    426: вскоре превратившаяся в гвардию кардинала. Напряженность отношений между
                    427: Ришелье и Анной Австрийской широко известна благодаря роману "Три
                    428: мушкетера". Гвардейцы кардинала носили красные плащи.
                    429: 
                    430: Источник:
                    431:    1. http://ru.wikipedia.org/wiki/Ришельё,_Арман_Жан_дю_Плесси
                    432:    2. http://ru.wikipedia.org/wiki/Гвардейцы_кардинала
                    433: 
                    434: Автор:
                    435: Мишель Матвеев (Санкт-Петербург)
                    436: 
                    437: Вопрос 23:
                    438: Один испанец услышал от индейцев легенду об острове БимИни, где бьет
                    439: чудесный источник, и снарядил экспедицию на его поиски. По словам
                    440: исследователя, команды на кораблях этой экспедиции были самыми ТАКИМИ в
                    441: морской истории. Какими - ТАКИМИ?
                    442: 
                    443: Ответ:
                    444: Старыми.
                    445: 
                    446: Комментарий:
                    447: Источник якобы возвращал пьющему молодость.
                    448: 
                    449: Источник:
                    450: Е.Н. Авадяева, Л.И. Зданович. 100 великих мореплавателей.
                    451: http://www.flibusta.net/b/177083/read
                    452: 
                    453: Автор:
                    454: Мишель Матвеев (Санкт-Петербург)
                    455: 
                    456: Вопрос 24:
                    457: Недолгой военной карьере известного человека положила конец австрийская
                    458: мина под ФоссАльтой. Назовите произведение, в котором он описал события,
                    459: произошедшие в его жизни непосредственно после этого.
                    460: 
                    461: Ответ:
                    462: "Прощай, оружие!".
                    463: 
                    464: Комментарий:
                    465: Этот роман Хемингуэя во многом автобиографичен.
                    466: 
                    467: Источник:
                    468: С.А. Мусский. 100 великих нобелевских лауреатов.
                    469: http://www.flibusta.net/b/94100/read
                    470: 
                    471: Автор:
                    472: Мишель Матвеев (Санкт-Петербург)
                    473: 
                    474: Вопрос 25:
                    475: Как начинается шуточный афоризм, который заканчивается так: "... тот
                    476: безнадежно отстал в плане вооружения"?
                    477: 
                    478: Ответ:
                    479: "Кто придет к нам с мечом...".
                    480: 
                    481: Зачет:
                    482: "Кто к нам с мечом придет...", "Кто с мечом к нам придет..."; "войдет",
                    483: "пойдет" и т.п. вместо "придет".
                    484: 
                    485: Комментарий:
                    486: Переделка знаменитой фразы "Кто к нам с мечом придет, от меча и
                    487: погибнет", которую произносит Александр Невский в одноименном фильме.
                    488: 
                    489: Источник:
                    490: http://lurkmore.to/Древнерусские_мемы
                    491: 
                    492: Автор:
                    493: Мишель Матвеев (Санкт-Петербург)
                    494: 
                    495: Вопрос 26:
                    496: Исследователь Кузьмин считает, что это было начало перевода иноязычного
                    497: произведения, поскольку ниже была оставлена целая страница свободного
                    498: места. Напишите это начало.
                    499: 
                    500: Ответ:
                    501: "О закрой свои бледные ноги".
                    502: 
                    503: Зачет:
                    504: "Обнажи свои бледные ноги"; "Протяни свои бледные ноги".
                    505: 
                    506: Комментарий:
                    507: Кузьмин полагает, что Брюсов начал переводить какое-то стихотворение,
                    508: оставив под продолжение перевода страницу записной книжки. Спустя
                    509: какое-то время он решил сделать эту строку одностишием.
                    510: 
                    511: Источник:
                    512: http://ru.wikipedia.org/wiki/О_закрой_свои_бледные_ноги
                    513: 
                    514: Автор:
                    515: Мишель Матвеев (Санкт-Петербург)
                    516: 
                    517: Вопрос 27:
                    518: Ученый Даниил Иткис заметил: "У нас нет наук обо всём красном, нет
                    519: разработок, объединяющих всё мягкое". Какое понятие, недавно ставшее
                    520: популярным, критиковал Иткис?
                    521: 
                    522: Ответ:
                    523: Нанотехнологии.
                    524: 
                    525: Комментарий:
                    526: Иткис считает, что и науки обо всём, что имеет размер порядка нескольких
                    527: нанометров, не очень логично объединять общим термином.
                    528: 
                    529: Источник:
                    530: http://www.gazeta.ru/education/2008/01/22_a_2587722.shtml
                    531: 
                    532: Автор:
                    533: Мишель Матвеев (Санкт-Петербург)
                    534: 
                    535: Вопрос 28:
                    536: В обсуждении шестой книги саги "Песнь Льда и Пламени" кто-то спросил,
                    537: как можно будет отправить ее другу, который отбывает тюремное
                    538: заключение. Ему ответили: "Не стоит беспокоиться. Ваш друг СДЕЛАЕТ ЭТО".
                    539: Ответьте двумя словами, что именно сделает.
                    540: 
                    541: Ответ:
                    542: Выйдет раньше.
                    543: 
                    544: Зачет:
                    545: Выйдет быстрее/скорее/первым.
                    546: 
                    547: Комментарий:
                    548: Ожидание книги сильно затягивается.
                    549: 
                    550: Источник:
                    551: http://www.anekdot.ru/id/733013/
                    552: 
                    553: Автор:
                    554: Мишель Матвеев (Санкт-Петербург)
                    555: 
                    556: Вопрос 29:
                    557: Финские оленеводы решили опрыскивать рога своих животных ТАКИМ спреем.
                    558: Ответьте составным словом: каким - ТАКИМ?
                    559: 
                    560: Ответ:
                    561: Светоотражающим.
                    562: 
                    563: Комментарий:
                    564: Олени часто попадают под автомобили, особенно в ночное время.
                    565: Светоотражающий спрей призван уменьшить вероятность аварии.
                    566: 
                    567: Источник:
                    568: http://ria.ru/earth/20140219/995820818.html
                    569: 
                    570: Автор:
                    571: Мишель Матвеев (Санкт-Петербург)
                    572: 
                    573: Вопрос 30:
                    574: Что в статье об одной научной гипотезе было названо хвостатым
                    575: инкубатором?
                    576: 
                    577: Ответ:
                    578: Комета.
                    579: 
                    580: Комментарий:
                    581: В статье обсуждалась гипотеза панспермИи. Согласно приводимому в статье
                    582: мнению, кометы - идеальные инкубаторы, в которых миллионы лет может
                    583: протекать так называемая пребиотическая эволюция.
                    584: 
                    585: Источник:
                    586: http://www.gazeta.ru/science/2007/08/16_a_2046400.shtml
                    587: 
                    588: Автор:
                    589: Мишель Матвеев (Санкт-Петербург)
                    590: 
                    591: Вопрос 31:
                    592: Во "Всеобщей истории, обработанной "Сатириконом"" битва при ПлатЕях
                    593: описывается так: "Оракулы предсказали поражение тому войску, которое
                    594: первым вступит в бой. Войска стали выжидать. Но спустя десять дней
                    595: раздался характерный треск. Это ПРОПУСК персов". Заполните пропуск.
                    596: 
                    597: Ответ:
                    598: Лопнуло терпение.
                    599: 
                    600: Комментарий:
                    601: Персы в конце концов атаковали и потерпели поражение. И дело не в
                    602: предсказании оракула, а в том, что греческое войско занимало хорошую
                    603: позицию для обороны.
                    604: 
                    605: Источник:
                    606: Всеобщая история, обработанная "Сатириконом".
                    607: http://www.flibusta.net/b/159772/read
                    608: 
                    609: Автор:
                    610: Мишель Матвеев (Санкт-Петербург)
                    611: 
                    612: Вопрос 32:
                    613: Интернет-статья называет ДирихлЕ ИМ. Дело в том, что Дирихле
                    614: сформулировал принцип Дирихле. Назовите ЕГО двумя словами.
                    615: 
                    616: Ответ:
                    617: Капитан Очевидность.
                    618: 
                    619: Комментарий:
                    620: Принцип Дирихле (если есть n+1 кроликов, которые рассажены в n клеток,
                    621: то хотя бы в одной клетке будет больше одного кролика) действительно
                    622: выглядит утверждением в стиле Капитана Очевидность, хотя бывает весьма
                    623: полезен. В формулировке вопроса мы тоже выступили в духе Капитана
                    624: Очевидность.
                    625: 
                    626: Источник:
                    627: http://lurkmore.to/Капитан_Очевидность
                    628: 
                    629: Автор:
                    630: Мишель Матвеев (Санкт-Петербург)
                    631: 
                    632: Вопрос 33:
                    633: В 1708 году графу Андрею Матвееву в Лондоне было нанесено оскорбление
                    634: арестом по жалобе кредитора, что вызвало возмущение в России. Считается,
                    635: что именно в связи с этим в Великобритании было впервые зафиксировано
                    636: понятие... Чего?
                    637: 
                    638: Ответ:
                    639: Дипломатического иммунитета.
                    640: 
                    641: Зачет:
                    642: Дипломатической неприкосновенности.
                    643: 
                    644: Комментарий:
                    645: Матвеев был официальным посланником Петра I. В связи с инцидентом
                    646: британский парламент принял акт о дипломатической неприкосновенности.
                    647: 
                    648: Источник:
                    649:    1. http://ru.wikipedia.org/wiki/Матвеев,_Андрей_Артамонович
                    650:    2. http://en.wikipedia.org/wiki/Diplomatic_immunity
                    651: 
                    652: Автор:
                    653: Мишель Матвеев (Санкт-Петербург)
                    654: 
                    655: Вопрос 34:
                    656: "Ну что, товарищи, воткнем капиталистам нашу иголочку?". Эти слова были
                    657: сказаны в 1964 году. Назовите эту "иголочку" двумя словами.
                    658: 
                    659: Ответ:
                    660: Останкинская телебашня.
                    661: 
                    662: Зачет:
                    663: Останкинская башня.
                    664: 
                    665: Комментарий:
                    666: Так высказался Хрущёв, приехавший на строительство Останкинской башни,
                    667: которое велось с 1960 по 1968 год.
                    668: 
                    669: Источник:
                    670: А.Ю. Хорошевский. 100 знаменитых символов советской эпохи.
                    671: http://www.flibusta.net/b/235474/read
                    672: 
                    673: Автор:
                    674: Мишель Матвеев (Санкт-Петербург)
                    675: 
                    676: Вопрос 35:
                    677: Когда Владимир Путин поздравлял российских волейболисток с победой на
                    678: чемпионате мира, Екатерина Гамова высказалась в том смысле, что путь к
                    679: победе был очень нелегким. Путин на это заметил, что третья часть только
                    680: начинается. Что это за третья часть?
                    681: 
                    682: Ответ:
                    683: Медные трубы.
                    684: 
                    685: Комментарий:
                    686: Гамова сказала, что волейболистки прошли огонь, воду и медные трубы.
                    687: Медные трубы соответствуют славе.
                    688: 
                    689: Источник:
                    690: http://www.sport-express.ru/newspaper/2006-11-18/1_5/
                    691: 
                    692: Автор:
                    693: Мишель Матвеев (Санкт-Петербург)
                    694: 
                    695: Вопрос 36:
                    696: Какие два слова одинаковой длины написаны с обратной стороны входной
                    697: двери в игровой центр "Gaga PlayLoft" [гАга плэйлОфт]?
                    698: 
                    699: Ответ:
                    700: Game over.
                    701: 
                    702: Комментарий:
                    703: Люди видят эти слова, когда, наигравшись, уходят из центра.
                    704: 
                    705: Источник:
                    706: Личные впечатления.
                    707: 
                    708: Автор:
                    709: Мишель Матвеев (Санкт-Петербург)
                    710: 
                    711: Тур:
                    712: 2 тур
                    713: 
                    714: Дата:
                    715: 13-Nov-2015
                    716: 
                    717: Редактор:
                    718: Алексей Полевой (Гомель) и Денис Рыбачук (Брест)
                    719: 
                    720: Инфо:
                    721: Редакторы благодарят за помощь в работе над пакетом команды "Ультиматум"
                    722: (Гомель), "Три обезьяны" (Брест), Дмитрия Спектора и команду "Фрейдзона"
                    723: (Гомель), Алексея Богословского и команду "Варяги" (а также Валерию
                    724: Аганину, Дарью Сесину, Виталию Кубрак и Наталью Гузеватую) (все -
                    725: Санкт-Петербург), Наиля Фарукшина (Навои), Николая Лёгенького
                    726: (Амстердам), Михаила Савченкова (Москва) и Евгения Миротина (Минск).
                    727: 
                    728: Вопрос 1:
                    729: Александр Ширвиндт, подчеркивая длительность сотрудничества с Михаилом
                    730: Державиным, упоминает ЕГО имени Грауэрмана. Назовите ЕГО.
                    731: 
                    732: Ответ:
                    733: Роддом.
                    734: 
                    735: Зачет:
                    736: Родильный дом.
                    737: 
                    738: Комментарий:
                    739: Григорий Львович Грауэрман - известный врач, акушер-гинеколог. Ширвиндт
                    740: считает, что их с Державиным совместный путь начался еще в роддоме,
                    741: несмотря на то что Державин родился на два года позднее. Но в том же
                    742: роддоме. Этот пакет тоже является плодом достаточно длительного
                    743: сотрудничества двух редакторов. :-)
                    744: 
                    745: Источник:
                    746: А.А. Ширвиндт. Восковые фигуры моего мини-музея.
                    747: http://rutracker.org/forum/viewtopic.php?t=5028191
                    748: 
                    749: Автор:
                    750: Денис Рыбачук (Брест)
                    751: 
                    752: Вопрос 2:
                    753: Для езидов змей - священное животное. В чем, по мнению езидов, он
                    754: однажды заткнул дыру?
                    755: 
                    756: Ответ:
                    757: В [Ноевом] ковчеге.
                    758: 
                    759: Зачет:
                    760: В дне [Ноева] ковчега.
                    761: 
                    762: Комментарий:
                    763: Религия езидов содержит элементы христианства. По их верованиям, однажды
                    764: Ноев ковчег сел на мель в горах, и днище получило пробоину. Змей свился
                    765: кольцами и течь заткнул.
                    766: 
                    767: Источник:
                    768: А. Кристи. Расскажи, как живешь. http://www.flibusta.net/b/29830/read
                    769: 
                    770: Автор:
                    771: Алексей Полевой (Гомель)
                    772: 
                    773: Вопрос 3:
                    774: Назовите город, в который вскоре после окончания Второй мировой войны
                    775: был отправлен в командировку художник-карикатурист Владимир ГАльба.
                    776: 
                    777: Ответ:
                    778: Нюрнберг.
                    779: 
                    780: Комментарий:
                    781: Освещать ход Нюрнбергского процесса надо было не только репортажами и
                    782: фотографиями, но и карикатурами.
                    783: 
                    784: Источник:
                    785: http://ru.wikipedia.org/wiki/Гальба,_Владимир_Александрович
                    786: 
                    787: Автор:
                    788: Алексей Полевой (Гомель)
                    789: 
                    790: Вопрос 4:
                    791: Карикатурист Владимир ГАльба называл сборник своих зарисовок и шаржей
                    792: выдуманным словом, отличающимся от реально существующего одной буквой.
                    793: Назовите это выдуманное слово.
                    794: 
                    795: Ответ:
                    796: "Гальбом".
                    797: 
                    798: Комментарий:
                    799: Каламбурил художник: у Гальбы - гальбомы.
                    800: 
                    801: Источник:
                    802: http://old-crocodile.livejournal.com/193702.html
                    803: 
                    804: Автор:
                    805: Денис Рыбачук (Брест)
                    806: 
                    807: Вопрос 5:
                    808: Героиню сериала, действие которого разворачивается в Британии в середине
                    809: прошлого века, хотят заставить бросить курить и поэтому иногда ругают за
                    810: нарушение ЕЕ. Назовите ЕЕ двухкоренным словом.
                    811: 
                    812: Ответ:
                    813: Светомаскировка.
                    814: 
                    815: Комментарий:
                    816: Светомаскировка во время второй мировой войны предполагала минимальное
                    817: использование света на случай бомбардировки. Курить, конечно, при этом
                    818: не запрещалось, но предосторожность в данном случае во благо.
                    819: 
                    820: Источник:
                    821: Сериал "Война Фойла", s01e03.
                    822: 
                    823: Автор:
                    824: Денис Рыбачук (Брест)
                    825: 
                    826: Вопрос 6:
                    827: Одну из росписей кукольного замка, созданного по заказу актрисы немого
                    828: кино Колин Мур, выполнил ОН. Назовите ЕГО фамилию.
                    829: 
                    830: Ответ:
                    831: Дисней.
                    832: 
                    833: Комментарий:
                    834: Дисней был художником, и актриса, возможно, знала его лично. Кстати,
                    835: символом компании "Уолт Дисней Пикчерз" является изображение замка.
                    836: 
                    837: Источник:
                    838: http://nnm.me/blogs/nurenberg/10-samyh-dorogih-detskih-igrushek/
                    839: 
                    840: Автор:
                    841: Алексей Полевой (Гомель)
                    842: 
                    843: Вопрос 7:
                    844: [Ведущему: прочитать вопрос чуть медленнее, чем обычно, - "под запись".]
                    845:    На одной юмористической карте морская держава Великобритания, которая
                    846: пытается влиять на Сенегал, Алжир, Ливию, Египет, Сомали, Сирию,
                    847: Афганистан и Норвегию, изображена в виде НЕГО. Назовите ЕГО.
                    848: 
                    849: Ответ:
                    850: Осьминог.
                    851: 
                    852: Комментарий:
                    853: Если подсчитать перечисленные в вопросе страны (кроме Великобритании),
                    854: то получится восемь - по одной на каждое щупальце.
                    855: 
                    856: Источник:
                    857: Документальный сериал "The Beauty of Maps" ("Очарование географических
                    858: карт"). http://rutracker.org/forum/viewtopic.php?t=4662390
                    859: 
                    860: Автор:
                    861: Денис Рыбачук (Брест)
                    862: 
                    863: Вопрос 8:
                    864: Словом "АЛЬФЫ" мы заменили пару слов.
                    865:    В книге "Галантный век" АЛЬФЫ называются колоннами, поддерживающими
                    866: храм любви. Какие АЛЬФЫ упоминаются в "Евгении Онегине"?
                    867: 
                    868: Ответ:
                    869: Стройные.
                    870: 
                    871: Комментарий:
                    872: Словом "АЛЬФЫ" заменены слова "женские ноги". Женщин в галантный век
                    873: обожествляли. "Три пары стройных женских ног" никак не мог отыскать
                    874: Пушкин. Слова "пара слов" - подсказка.
                    875: 
                    876: Источник:
                    877:    1. Э. Фукс. Галантный век. - М.: Республика, 1994.
                    878:    2. А.С. Пушкин. Евгений Онегин.
                    879: http://www.rvb.ru/pushkin/01text/04onegin/01onegin/0836.htm
                    880: 
                    881: Автор:
                    882: Денис Рыбачук (Брест)
                    883: 
                    884: Вопрос 9:
                    885: В произведении Торнтона Уайлдера один театр часто ставил новые пьесы,
                    886: так как ОН отлично справлялся со своими обязанностями. Подскажем, что
                    887: ЕГО должность и сегодня сохранена в некоторых театрах. Назовите ЕГО.
                    888: 
                    889: Ответ:
                    890: Суфлер.
                    891: 
                    892: Комментарий:
                    893: Репертуар постоянно обновлялся, и актеры могли позволить себе не твердо
                    894: заучивать текст, так как суфлер всегда приходил на помощь. Слово
                    895: "подскажем" - небольшая подсказка.
                    896: 
                    897: Источник:
                    898:    1. Т. Уайлдер. Мост короля Людовика Святого.
                    899: http://www.flibusta.net/b/170994/read
                    900:    2. http://ru.wikipedia.org/wiki/Суфлёр
                    901: 
                    902: Автор:
                    903: Денис Рыбачук (Брест)
                    904: 
                    905: Вопрос 10:
                    906: В 1522 году доктору Вертту пришлось СДЕЛАТЬ ЭТО, чтобы провести научные
                    907: наблюдения во время родов. По расхожему мнению, однажды ЭТО СДЕЛАЛ
                    908: человек, умерший в 1970 году в Нью-Йорке. Назовите этого человека.
                    909: 
                    910: Ответ:
                    911: [Александр Федорович] Керенский.
                    912: 
                    913: Комментарий:
                    914: Мужчины на роды в то время не допускались, и Вертту пришлось переодеться
                    915: в женскую одежду. По одной из версий, Керенскому тоже пришлось надеть на
                    916: себя платье, что впоследствии он отрицал.
                    917: 
                    918: Источник:
                    919:    1. Г. Хаггард. От знахаря до врача. История науки врачевания.
                    920: http://www.loveread.ec/read_book.php?id=44533&p=7
                    921:    2. http://ru.wikipedia.org/wiki/Керенский,_Александр_Фёдорович
                    922: 
                    923: Автор:
                    924: Денис Рыбачук (Брест)
                    925: 
                    926: Вопрос 11:
                    927: ОН является объектом сделки в известной повести. Можно сказать, что
                    928: перед своим первым публичным выступлением Марк Твен купил ЕГО. Назовите
                    929: ЕГО.
                    930: 
                    931: Ответ:
                    932: Смех.
                    933: 
                    934: Комментарий:
                    935: Чтобы гарантировать успех своих острот, юморист посадил в зале человека
                    936: с заразительным смехом. В повести Джеймса Крюса смех продают.
                    937: 
                    938: Источник:
                    939:    1. http://ru.wikipedia.org/wiki/Тим_Талер,_или_Проданный_смех
                    940:    2. М. Твен. Налегке. http://www.flibusta.net/b/204073/read
                    941: 
                    942: Автор:
                    943: Денис Рыбачук (Брест)
                    944: 
                    945: Вопрос 12:
                    946: Герой "Записок кавалериста", уходя от обстрела, проклинает тех, кто
                    947: СДЕЛАЛ ЭТО. Древние скандинавы верили, что людей научил ДЕЛАТЬ ЭТО бог
                    948: плодородия Фрейр. Что делать?
                    949: 
                    950: Ответ:
                    951: Пахать землю.
                    952: 
                    953: Зачет:
                    954: По ключевому слову "пахать" или однокоренным глаголам.
                    955: 
                    956: Комментарий:
                    957: Из названия ясно, что герой уходил от обстрела на лошади. Скакать по
                    958: вспаханному полю трудно.
                    959: 
                    960: Источник:
                    961:    1. Н.С. Гумилёв. Записки кавалериста.
                    962: http://www.flibusta.net/b/21684/read
                    963:    2. http://www.mifs.su/gods/freir.html
                    964: 
                    965: Автор:
                    966: Денис Рыбачук (Брест)
                    967: 
                    968: Вопрос 13:
                    969: Персонаж Юза Алешковского гарантирует свежесть дорогого продукта и
                    970: предлагает для проверки поместить небольшое его количество в ИКС на пару
                    971: дней. "ИКС" появился в 1972 году. Какое слово мы заменили на ИКС?
                    972: 
                    973: Ответ:
                    974: Аквариум.
                    975: 
                    976: Комментарий:
                    977: Мол, икра такая свежая, что если поместить ее в аквариум, то скоро
                    978: появятся мальки. Конечно, на самом деле этого не произойдет. Во втором
                    979: случае речь шла о музыкальной группе.
                    980: 
                    981: Источник:
                    982:    1. Ю. Алешковский. Рука. http://www.flibusta.net/b/106459/read
                    983:    2. http://ru.wikipedia.org/wiki/Аквариум_(группа)
                    984: 
                    985: Автор:
                    986: Денис Рыбачук (Брест)
                    987: 
                    988: Вопрос 14:
                    989: Осматривая Аквариум Берлинского зоопарка, персонаж Набокова называет ИКС
                    990: вековым куполом. ИКС состоит из двух щитов. Какие два слова мы заменили
                    991: на ИКС?
                    992: 
                    993: Ответ:
                    994: Панцирь черепахи.
                    995: 
                    996: Комментарий:
                    997: Черепахи - известные долгожители. Панцирь черепахи состоит из спинного и
                    998: брюшного щитов. Кстати, римское воинское построение "черепаха" тоже
                    999: составлялось из щитов.
                   1000: 
                   1001: Источник:
                   1002:    1. В.В. Набоков. Путеводитель по Берлину.
                   1003: http://www.flibusta.net/b/38645/read
                   1004:    2. http://rediskadowell.blogspot.com.by/p/zhukovsky-viktor.html
                   1005: 
                   1006: Автор:
                   1007: Денис Рыбачук (Брест)
                   1008: 
                   1009: Вопрос 15:
                   1010: В одном фильме снайпер рассказывает об убитом ночью немце. Фразу,
                   1011: которая звучит далее, можно увидеть на АЛЬФАХ. Назовите АЛЬФУ двумя
                   1012: словами.
                   1013: 
                   1014: Ответ:
                   1015: Пачка сигарет.
                   1016: 
                   1017: Зачет:
                   1018: Пачка папирос; реклама сигарет.
                   1019: 
                   1020: Комментарий:
                   1021: Снайпер прицелился ночью по огоньку закуриваемой сигареты. Выслушав
                   1022: доклад, еще один персонаж фильма констатирует: "Курение убивает". Так
                   1023: что те, кто в одном из предыдущих вопросов пытались отучить девушку
                   1024: курить, были абсолютно правы.
                   1025: 
                   1026: Источник:
                   1027: Х/ф "Битва за Севастополь" (2015), реж. Сергей Мокрицкий.
                   1028: 
                   1029: Автор:
                   1030: Денис Рыбачук (Брест)
                   1031: 
                   1032: Вопрос 16:
                   1033:    <раздатка>
                   1034:    das touch
                   1035:    </раздатка>
                   1036:    Пиво, сваренное по старинному рецепту, обладает приятным золотистым
                   1037: цветом. Какие две буквы в его названии мы пропустили?
                   1038: 
                   1039: Ответ:
                   1040: Mi.
                   1041: 
                   1042: Комментарий:
                   1043: Пиво, восстановленное по остаткам на стенках кувшина, найденного в
                   1044: гробнице древнего царя Мидаса, получило название "Midas touch" -
                   1045: "Прикосновение Мидаса". Как известно, от прикосновения Мидаса вещи
                   1046: становились золотыми.
                   1047: 
                   1048: Источник:
                   1049: http://nnm.me/blogs/Le0nidy4/7-drevnih-alkogolnyh-napitkov-recepty-kotoryh-udalos-vosstanovit/
                   1050: 
                   1051: Автор:
                   1052: Алексей Полевой (Гомель)
                   1053: 
                   1054: Вопрос 17:
                   1055: Йеменский город Шибам исторически застраивался внутри крепостных стен,
                   1056: поэтому Шибам иногда называют городом ИХ. С НИМ в произведении Фрица
                   1057: Лейбера сравнивается выигрыш удачливого игрока. Назовите ЕГО.
                   1058: 
                   1059: Ответ:
                   1060: Небоскреб.
                   1061: 
                   1062: Комментарий:
                   1063: Шибам славится своей высотной архитектурой, ограниченный в пространстве
                   1064: город рос ввысь. Выигранные фишки игрок собирает в высокий
                   1065: столбик-небоскреб.
                   1066: 
                   1067: Источник:
                   1068:    1. http://ru.wikipedia.org/wiki/Шибам
                   1069:    2. Ф. Лейбер. Побросаю-ка я кости.
                   1070: http://www.flibusta.net/b/69868/read
                   1071: 
                   1072: Автор:
                   1073: Денис Рыбачук (Брест)
                   1074: 
                   1075: Вопрос 18:
                   1076: На расположенной в Иране Бехистунской скале сохранилась надпись,
                   1077: благодаря чему скалу часто сравнивают с НИМ. Назовите ЕГО двумя словами.
                   1078: 
                   1079: Ответ:
                   1080: Розеттский камень.
                   1081: 
                   1082: Комментарий:
                   1083: Бехистунская надпись была нанесена по приказу царя Дария I о событиях
                   1084: 523-521 гг. до н.э. и является одним из крупнейших эпиграфических
                   1085: памятников древности. Дешифровка надписи положила начало расшифровке
                   1086: клинописного письма многих народов древнего Востока, как и дешифровка
                   1087: надписи на Розеттском камне положила начало расшифровке Египетских
                   1088: иероглифов.
                   1089: 
                   1090: Источник:
                   1091:    1. http://top10x.ru/top-10-interesnyie-i-udivitelnyie-arheologicheskie-nahodki/
                   1092:    2. http://ru.wikipedia.org/wiki/Бехистунская_надпись
                   1093: 
                   1094: Автор:
                   1095: Алексей Полевой (Гомель)
                   1096: 
                   1097: Вопрос 19:
                   1098: Надпись на Бехистунской скале, которая позволила расшифровать клинопись,
                   1099: высечена на трех камнях. Какое слово в вопросе мы заменили?
                   1100: 
                   1101: Ответ:
                   1102: Языках.
                   1103: 
                   1104: Комментарий:
                   1105: Бехистунская надпись (как и надпись на Розеттском камне) является
1.4     ! rubashki 1106: трехъязычной, что несколько облегчило расшифровку.
1.1       rubashki 1107: 
                   1108: Источник:
                   1109: http://ru.wikipedia.org/wiki/Бехистунская_надпись
                   1110: 
                   1111: Автор:
                   1112: Алексей Полевой (Гомель)
                   1113: 
                   1114: Вопрос 20:
                   1115: Ответ на прошлый вопрос может помочь вам ответить на этот.
                   1116:    Артем Кречетников пишет, что жители города кричали "Всплывай!", но
                   1117: из-за золотых и серебряных украшений ОН утонул. Сейчас этот город
                   1118: является столицей. Назовите этот город.
                   1119: 
                   1120: Ответ:
                   1121: Киев.
                   1122: 
                   1123: Комментарий:
                   1124: Надеемся, что предыдущий ответ "Язык" довел до Киева. Он - это
                   1125: сброшенный в воду идол Перуна. Неспособность всплыть была отнесена к
                   1126: слабости старых богов и силе христианского. Похожая история происходила
                   1127: и в Новгороде, но столицей он не является.
                   1128: 
                   1129: Источник:
                   1130: http://nnm.me/blogs/dendy2008/kievskiy-knyaz-vladimir-svyatoy-i-politicheskaya-zloba-dnya/
                   1131: 
                   1132: Автор:
                   1133: Алексей Полевой (Гомель)
                   1134: 
                   1135: Вопрос 21:
                   1136: В рассказе Владимира Набокова антрепренер перед самым началом концерта
                   1137: ищет отказавшегося выступать музыканта, но вскоре понимает, что надо
                   1138: спасать ЕГО. Назовите ЕГО словом с удвоенной согласной.
                   1139: 
                   1140: Ответ:
                   1141: Кассир.
                   1142: 
                   1143: Комментарий:
                   1144: Зрители, не дождавшись выступления, взяли бы кассу штурмом. Кассиру
                   1145: могло достаться...
                   1146: 
                   1147: Источник:
                   1148: В.В. Набоков. Бахман. http://www.flibusta.net/b/158409/read
                   1149: 
                   1150: Автор:
                   1151: Денис Рыбачук (Брест)
                   1152: 
                   1153: Вопрос 22:
                   1154: [Ведущему: никак не показывать, что один из "арманьяков" написан с
                   1155: заглавной буквы, а другой - нет.]
                   1156:    По одной из версий, арманьЯк не достиг популярности коньякА потому,
                   1157: что у АрманьЯка не было ЕГО и из-за этого экспорт не был столь велик.
                   1158: Назовите ЕГО тремя словами.
                   1159: 
                   1160: Ответ:
                   1161: Выход к морю.
                   1162: 
                   1163: Комментарий:
                   1164: Как и коньяк, арманьяк получил свое название в честь местности, в
                   1165: которой производился. Во втором случае речь идет о том, чего не хватало
                   1166: графству Арманьяк для успешного экспорта. Если из графства Коньяк выход
                   1167: к морю был легко доступен, то графство Арманьяк было расположено хуже
                   1168: для экспорта.
                   1169: 
                   1170: Источник:
                   1171: http://lenta.ru/articles/2015/08/07/armagnac/
                   1172: 
                   1173: Автор:
                   1174: Денис Рыбачук (Брест)
                   1175: 
                   1176: Вопрос 23:
                   1177: (pic: 20150816.jpg)
                   1178:    Долгое время ОН был основным источником дохода Бахрейна, а сейчас
                   1179: музей ЕГО является одной из главных достопримечательностей страны.
                   1180: Назовите ЕГО.
                   1181: 
                   1182: Ответ:
                   1183: Жемчуг.
                   1184: 
                   1185: Комментарий:
                   1186: Бахрейн (в отличие от Арманьяка) имеет выход к морю, т.е. к Персидскому
                   1187: заливу, где и добывается жемчуг. Слово "жемчужина" можно употребить в
                   1188: смысле "самое важное и главное, что стоит увидеть". На фотографии -
                   1189: музей жемчуга в Манаме, который венчает "жемчужина".
                   1190: 
                   1191: Источник:
                   1192: "National Geographic. Россия", N 136, январь 2015 г. - С. 47.
                   1193: 
                   1194: Автор:
                   1195: Алексей Полевой (Гомель)
                   1196: 
                   1197: Вопрос 24:
                   1198: Рассказывая о скученности людей в больничных палатах начала XVIII века,
                   1199: Амбруаз Паре употребляет слово "король". Какое слово мы заменили словом
                   1200: "король"?
                   1201: 
                   1202: Ответ:
                   1203: Валет.
                   1204: 
                   1205: Комментарий:
                   1206: Паре - француз, и слово "валет" также французское - пациентов укладывали
                   1207: на кровати "валетом", т.е. по двое на кровать, головами в разные
                   1208: стороны.
                   1209: 
                   1210: Источник:
                   1211: Г. Хаггард. От знахаря до врача. История науки врачевания.
                   1212: http://www.loveread.ec/read_book.php?id=44533&p=7
                   1213: 
                   1214: Автор:
                   1215: Денис Рыбачук (Брест)
                   1216: 
                   1217: Вопрос 25:
                   1218: После Второй мировой войны отношения Японии и Китая строились по формуле
                   1219: "Нолики в политике, крестики в экономике". Что мы заменили на
                   1220: "крестики-нолики"?
                   1221: 
                   1222: Ответ:
                   1223: Горячо-холодно.
                   1224: 
                   1225: Зачет:
                   1226: Горячо, холодно (в любом порядке).
                   1227: 
                   1228: Комментарий:
                   1229: Если в политике Япония относилась к Китаю очень прохладно, то в качестве
                   1230: рынка для своих товаров и дешевой рабочей силы Китай очень
                   1231: приветствовался, а Китай не гнушался получать японские инвестиции.
                   1232: "Горячо-холодно" и "Крестики-нолики" - названия игр.
                   1233: 
                   1234: Источник:
                   1235: http://nnm.me/blogs/atck/uroki-nenavisti/
                   1236: 
                   1237: Автор:
                   1238: Алексей Полевой (Гомель)
                   1239: 
                   1240: Вопрос 26:
                   1241: Герой повести "Пилот первого класса" после десятилетий службы начинает
                   1242: учить ЕЕ наизусть. Назовите ЕЕ.
                   1243: 
                   1244: Ответ:
                   1245: Таблица проверки [остроты] зрения.
                   1246: 
                   1247: Зачет:
                   1248: Таблица Сивцева; таблица Головина-Сивцева.
                   1249: 
                   1250: Комментарий:
                   1251: С возрастом у пилота падает зрение, но бросать небо он не хочет. Чтобы
                   1252: пройти ежегодную комиссию, летчик идет на некоторые хитрости.
                   1253: 
                   1254: Источник:
                   1255: В.В. Кунин. Пилот первого класса. http://www.flibusta.net/b/67439/read
                   1256: 
                   1257: Автор:
                   1258: Денис Рыбачук (Брест)
                   1259: 
                   1260: Вопрос 27:
                   1261: Марк Твен некоторое время прожил на Гавайских островах. Во время
                   1262: последующего путешествия в Европу писатель сравнил ЕГО с небольшой
                   1263: кастрюлькой. Назовите ЕГО.
                   1264: 
                   1265: Ответ:
                   1266: Везувий.
                   1267: 
                   1268: Комментарий:
                   1269: Кастрюлька - это потому, что оттуда вырывается пар. Вулканы на Гавайях
                   1270: масштабней, поэтому Везувий Твена явно не впечатлил.
                   1271: 
                   1272: Источник:
                   1273: М. Твен. Налегке. http://www.flibusta.net/b/204073/read
                   1274: 
                   1275: Автор:
                   1276: Денис Рыбачук (Брест)
                   1277: 
                   1278: Вопрос 28:
                   1279: Однажды в матче между актерскими футбольными командами принял участие
                   1280: известный футболист Игорь Нетто. Рассказывая об этом, Александр Ширвиндт
                   1281: упоминает прекрасную работу ЕГО. Назовите ЕГО.
                   1282: 
                   1283: Ответ:
                   1284: Гример.
                   1285: 
                   1286: Комментарий:
                   1287: Конечно, в матче актерских футбольных команд профессионалу играть было
                   1288: нельзя. И Нетто загримировали до неузнаваемости.
                   1289: 
                   1290: Источник:
                   1291: А.А. Ширвиндт. Восковые фигуры моего мини-музея.
                   1292: http://rutracker.org/forum/viewtopic.php?t=5028191
                   1293: 
                   1294: Автор:
                   1295: Денис Рыбачук (Брест)
                   1296: 
                   1297: Вопрос 29:
                   1298: Персонаж Грэма Грина работает врачом и легко понимает собеседника,
                   1299: который говорит невнятно и нечленораздельно. Какое слово в вопросе мы
                   1300: пропустили?
                   1301: 
                   1302: Ответ:
                   1303: Зубным.
                   1304: 
                   1305: Комментарий:
                   1306: "... Этот приглушенный, нечленораздельный вопрос мог понять только
                   1307: дантист...".
                   1308: 
                   1309: Источник:
                   1310: Г. Грин. Сила и слава. http://www.flibusta.net/b/188831/read
                   1311: 
                   1312: Автор:
                   1313: Алексей Полевой (Гомель)
                   1314: 
                   1315: Вопрос 30:
                   1316: Для митраизма были характерны церемонии жертвоприношения. Так как
                   1317: митраизм был распространен и в Южной Европе, то, по одной из версий, в
                   1318: ЕЕ основе лежит ритуал. Назовите ЕЕ.
                   1319: 
                   1320: Ответ:
                   1321: Коррида.
                   1322: 
                   1323: Комментарий:
                   1324: Испания - южная страна, а в жертву приносили быка.
                   1325: 
                   1326: Источник:
                   1327: http://nnm.me/blogs/atck/5-ischeznuvshih-religiy/
                   1328: 
                   1329: Автор:
                   1330: Алексей Полевой (Гомель)
                   1331: 
                   1332: Вопрос 31:
                   1333: Персонаж сериала, действие которого происходит в годы Второй мировой
                   1334: войны, жалеет, что отцовское ЭТО унаследовал не он, а его брат. Назовите
                   1335: ЭТО двухкоренным словом.
                   1336: 
                   1337: Ответ:
                   1338: Плоскостопие.
                   1339: 
                   1340: Комментарий:
                   1341: Брату повезло - он не попал на войну, будучи признан негодным к службе.
                   1342: 
                   1343: Источник:
                   1344: Сериал "Война Фойла", s07e01.
                   1345: 
                   1346: Автор:
                   1347: Денис Рыбачук (Брест)
                   1348: 
                   1349: Вопрос 32:
                   1350: В одном из римских аэропортов можно увидеть стилизованный земной шар,
                   1351: внутри которого находится человек. Какое слово в вопросе мы пропустили?
                   1352: 
                   1353: Ответ:
                   1354: Витрувианский.
                   1355: 
                   1356: Комментарий:
                   1357: Вся композиция стилизована под известный рисунок Леонардо. Римский
                   1358: аэропорт носит имя Леонадро да Винчи.
                   1359: 
                   1360: Источник:
                   1361: http://ftour.otzyv.ru/read.php?id=157192
                   1362: 
                   1363: Автор:
                   1364: Алексей Полевой (Гомель)
                   1365: 
                   1366: Вопрос 33:
                   1367: [Ведущему: в предложении "Кстати, готовить она любила" интонационное
                   1368: ударение на слово "готовить".]
                   1369:    Недавно, к 125-летию со дня ЕЕ рождения, был издан сборник кулинарных
                   1370: рецептов, причем к каждому блюду был указан дополнительный "ингредиент",
                   1371: который можно и не добавлять. Кстати, готовить она любила. Назовите ЕЕ.
                   1372: 
                   1373: Ответ:
                   1374: [Агата] Кристи.
                   1375: 
                   1376: Комментарий:
                   1377: Сборник включал отравленные блюда из произведений Агаты Кристи. К
                   1378: каждому из блюд был указан и примененный в детективе яд. В отличие от
                   1379: мытья посуды, которое Агата ненавидела, готовить она любила.
                   1380: 
                   1381: Источник:
                   1382: http://radiovesti.ru/article/show/article_id/170084
                   1383: 
                   1384: Автор:
                   1385: Алексей Полевой (Гомель)
                   1386: 
                   1387: Вопрос 34:
                   1388: Героиня Владимира Кунина работает на рынке, в том числе потому, что там
                   1389: хорошая акустика. Во время ее работы ИКС алчно раскрыт. ИКС есть в
                   1390: названии рассказа. Напишите это название.
                   1391: 
                   1392: Ответ:
                   1393: "Человек в футляре".
                   1394: 
                   1395: Комментарий:
                   1396: Персонаж зарабатывает на рынке игрой на гитаре. В алчно раскрытый футляр
                   1397: от гитары прохожие кидают деньги. Кроме хорошей акустики, причиной явно
                   1398: является то, что на рынке много людей.
                   1399: 
                   1400: Источник:
                   1401:    1. В.В. Кунин. Русские на Мариенплац.
                   1402: http://www.flibusta.net/b/93538/read
                   1403:    2. http://ru.wikipedia.org/wiki/Человек_в_футляре
                   1404: 
                   1405: Автор:
                   1406: Денис Рыбачук (Брест)
                   1407: 
                   1408: Вопрос 35:
                   1409: Полотна Ван Гога часто служили объектами для критики. В качестве чего
                   1410: некий господин Ванель использовал картину Ван Гога?
                   1411: 
                   1412: Ответ:
                   1413: Мишень.
                   1414: 
                   1415: Зачет:
                   1416: По слову "мишень" с различными уточнениями, например, "мишень для
                   1417: стрельбы", "мишень для дартса".
                   1418: 
                   1419: Комментарий:
                   1420: Картины стали мишенью не только для критиков, но и для некоторых
                   1421: стрелков. Картина досталась Ванелю, когда еще не представляла большой
                   1422: ценности.
                   1423: 
                   1424: Источник:
                   1425: А. Перрюшо. Жизнь Ван Гога. http://www.flibusta.net/b/42235/read
                   1426: 
                   1427: Автор:
                   1428: Алексей Полевой (Гомель)
                   1429: 
                   1430: Вопрос 36:
                   1431: Литератор Андрей Седых начинал свой творческий путь в 1918 году в
                   1432: редколлегии журнала "Мы". В 1973 году стал главным редактором газеты
                   1433: "Новое русское слово" и еще долго трудился на этом поприще. Умер в 1994
                   1434: году. Какой жанр, согласно Александру Генису, наиболее хорошо удавался
                   1435: Седых?
                   1436: 
                   1437: Ответ:
                   1438: Некролог.
                   1439: 
                   1440: Комментарий:
                   1441: Андрей Седых был редкостным долгожителем, и ему довелось написать
                   1442: некрологи своим друзьям, коллегам и даже ученикам. Вопрос также написан
                   1443: помпезным и заштампованным языком некролога. Наш пакет начинался с
                   1444: роддома, поэтому такое завершение, хотя и помпезно, но вполне логично.
                   1445: 
                   1446: Источник:
                   1447:    1. А.А. Генис. Довлатов и окрестности.
                   1448: http://www.flibusta.net/b/354275/read
                   1449:    2. http://ru.wikipedia.org/wiki/Андрей_Седых
                   1450: 
                   1451: Автор:
                   1452: Денис Рыбачук (Брест)
                   1453: 
                   1454: Тур:
                   1455: 3 тур
                   1456: 
                   1457: Дата:
                   1458: 18-Dec-2015
                   1459: 
                   1460: Редактор:
                   1461: Александр Пономарёв (Пермь)
                   1462: 
                   1463: Инфо:
                   1464: При подготовке пакета вопросов ценные замечания дали: Виктор Абдураманов
                   1465: (Березники), Алексей Гилёв (Пермь), Арсений Глазовский (Ярославль),
                   1466: Виктор Дзекановский (Санкт-Петербург), Тимур Кафиатуллин (Москва),
                   1467: Константин Костенко (Гремячинск - Пермь), Сергей Лобачёв (Нижний
                   1468: Новгород), Илья Иванов (Пермь), Мишель Матвеев (Санкт-Петербург), Айрат
                   1469: Мухарлямов (Казань), Александр Печёный (Пермь - Москва), Станислав
                   1470: Прошин (Екатеринбург), Алена Тарасова (Санкт-Петербург - Лондон),
                   1471: Григорий Теплых (Пермь), Елизавета Шестакова (Снежинск - Челябинск),
                   1472: Александр Шишкин (Верещагино - Пермь), Юрий Югов (Звездный - Пермь),
                   1473: команды Пермского центра "Муравейник".
                   1474: 
                   1475: Вопрос 1:
                   1476: У бодибилдера Тони Фримена были очень широкие плечи, узкая талия и
                   1477: громадные бедра. Поэтому его прозвище начиналось с ИКСА. Назовите ИКС.
                   1478: 
                   1479: Ответ:
                   1480: Икс.
                   1481: 
                   1482: Зачет:
                   1483: X.
                   1484: 
                   1485: Комментарий:
                   1486: Фигура Тони Фримена схожа с буквой Х [икс], отсюда прозвище "Икс-Мэн".
                   1487: 
                   1488: Источник:
                   1489: http://www.builderbody.ru/toni-frimen-figura-fantasticheskogo-masshtaba/
                   1490: 
                   1491: Автор:
                   1492: Максим Драчёв (Кемерово)
                   1493: 
                   1494: Вопрос 2:
                   1495: Тем, кто боится СДЕЛАТЬ ЭТО, рекомендуется одеваться ярко. Фраза со
                   1496: значением "Убирайся вон" может буквально переводиться с испанского как
                   1497: "Купи себе лес и СДЕЛАЙ ЭТО в нем". Что мы заменили словами "СДЕЛАТЬ
                   1498: ЭТО"?
                   1499: 
                   1500: Ответ:
                   1501: Заблудиться.
                   1502: 
                   1503: Зачет:
                   1504: Потеряться.
                   1505: 
                   1506: Комментарий:
                   1507: Если вы заблудитесь в яркой одежде, найти вас сверху будет чуть легче.
                   1508: 
                   1509: Источник:
                   1510:    1. http://www.nexplorer.ru/news__11365.htm
                   1511:    2. http://www.adme.ru/svoboda-kultura/kupi-sebe-les-i-zabludis-v-nem-i-esche-20-idiom-so-vsego-mira-860410/
                   1512: 
                   1513: Автор:
                   1514: Игорь Демьянцев (Гомель)
                   1515: 
                   1516: Вопрос 3:
                   1517: Формально главнокомандующим являлся великий князь Николай Николаевич, но
                   1518: недалеко от штаба была ставка императора, при котором были канцлер,
                   1519: военный министр и шеф жандармов. Кроме того, союзная румынская армия
                   1520: подчинялась собственному командующему. При описании этой ситуации Борис
                   1521: Акунин упомянул российского двуглавого орла и нескольких других
                   1522: животных. Перечислите их.
                   1523: 
                   1524: Ответ:
                   1525: Лебедь, щука и рак.
                   1526: 
                   1527: Комментарий:
                   1528: Каждый тянул в свою сторону.
                   1529: 
                   1530: Источник:
                   1531: Б. Акунин. Турецкий гамбит. http://www.flibusta.is/b/305713/read
                   1532: 
                   1533: Автор:
                   1534: Григорий Теплых (Пермь)
                   1535: 
                   1536: Вопрос 4:
                   1537: После того как основатель азербайджанского театра Гусейн Араблинский
                   1538: изменил внешность, его отношения с семьей ухудшились. Родственников даже
                   1539: больше разозлило не само изменение, а то, что Гусейн начал после этого
                   1540: делать в театре. Что именно он начал делать?
                   1541: 
                   1542: Ответ:
                   1543: Играть женские роли.
                   1544: 
                   1545: Зачет:
                   1546: Синонимичные ответы.
                   1547: 
                   1548: Комментарий:
                   1549: Он сбрил усы. Женские роли лучше играть без них. Его
                   1550: родственники-мусульмане были не только против уничтожения усов, но и
                   1551: возмущены исполнением женских ролей.
                   1552: 
                   1553: Источник:
                   1554: http://portal.azertag.az/ru/node/1900
                   1555: 
                   1556: Автор:
                   1557: Русиф Гусейнов (Баку)
                   1558: 
                   1559: Вопрос 5:
                   1560: В фильме "Нападение помидоров-убийц" замаскированный агент выдал себя,
                   1561: когда попросил емкость с НИМ. Назовите ЕГО одним словом.
                   1562: 
                   1563: Ответ:
                   1564: Кетчуп.
                   1565: 
                   1566: Комментарий:
                   1567: Дело было во время приема пищи. Человечество вело войну с помидорами, и
                   1568: человеческий агент был как раз в костюме помидора.
                   1569: 
                   1570: Источник:
                   1571: Х/ф "Нападение помидоров-убийц" (1978), реж. Джон Де Белло.
                   1572: 
                   1573: Автор:
                   1574: Григорий Теплых (Пермь)
                   1575: 
                   1576: Вопрос 6:
                   1577: На плакатах музыкальной группы "Тролль гнёт ель" ОНИ действительно
                   1578: похожи на половинки елового дерева. Чем одна из НИХ отличается от
                   1579: другой?
                   1580: 
                   1581: Ответ:
                   1582: Точками над Ё.
                   1583: 
                   1584: Зачет:
                   1585: Точками над Е; точками.
                   1586: 
                   1587: Комментарий:
                   1588: ОНИ - буквы Е и Ё. На плакатах их короткие линии не расположены
                   1589: горизонтально, а наклонены под углом, как еловые ветки.
                   1590: 
                   1591: Источник:
                   1592: http://ru.wikipedia.org/wiki/Тролль_гнёт_ель
                   1593: 
                   1594: Автор:
                   1595: Александр Пономарёв (Пермь)
                   1596: 
                   1597: Вопрос 7:
                   1598: В оккультизме демон Бельфегор предлагает людям хитроумные изобретения и
                   1599: таким образом способствует техническому прогрессу человечества. Каким
                   1600: человеческим недостатком, по словам Питера Бинсфельда, Бельфегор
                   1601: пользуется для соблазнения людей?
                   1602: 
                   1603: Ответ:
                   1604: Ленью.
                   1605: 
                   1606: Комментарий:
                   1607: Чтобы не работали.
                   1608: 
                   1609: Источник:
                   1610: http://ru.wikipedia.org/wiki/Бельфегор
                   1611: 
                   1612: Автор:
                   1613: Игорь Демьянцев (Гомель), Мишель Матвеев (Санкт-Петербург) - написан
                   1614: независимо
                   1615: 
                   1616: Вопрос 8:
                   1617: (pic: 20150817.jpg)
                   1618:    Мост в Стоктоне назван в ЕЕ честь. Назовите ЕЕ словом с двумя буквами
                   1619: "о".
                   1620: 
                   1621: Ответ:
                   1622: Бесконечность.
                   1623: 
                   1624: Комментарий:
                   1625: Отражаясь в воде, мост напоминает знак бесконечности. На розданном
                   1626: изображении начало и конец моста обрезаны. Из двух "о" тоже можно
                   1627: составить знак бесконечности.
                   1628: 
                   1629: Источник:
                   1630: http://www.adme.ru/svoboda-puteshestviya/25-nerealnyh-mostov-766760/
                   1631: 
                   1632: Автор:
                   1633: Дмитрий Богданов (Белгород)
                   1634: 
                   1635: Вопрос 9:
                   1636: Кем будущая императрица Елизавета Петровна многократно становилась с
                   1637: целью усиления влияния среди гвардейских офицеров?
                   1638: 
                   1639: Ответ:
                   1640: Крестной матерью.
                   1641: 
                   1642: Зачет:
                   1643: Крестной матерью детей; крестной.
                   1644: 
                   1645: Комментарий:
                   1646: Елизавета становилась крестной матерью для детей гвардейских офицеров,
                   1647: что очень сближало их с принцессой и будущей правительницей.
                   1648: 
                   1649: Источник:
                   1650:    1. Д/ф "Елизавета Петровна. Путь на трон" (2012).
                   1651: http://www.redmediatv.ru/history/gosudarstvennye-perevoroty-v-rossii/elizaveta-petrovna-put-na-tron.html
                   1652:    2. http://ruskline.ru/history/2013/12/07/vy_znaete_chya_ya_doch_stupajte_za_mnoj/
                   1653: 
                   1654: Автор:
                   1655: Евгений Котович (Кишинев)
                   1656: 
                   1657: Вопрос 10:
                   1658: По словам Солженицына, в лагере вознаграждение за сотрудничество с
                   1659: властями пересылалось осведомителю почтовым переводом. Поэтому если
                   1660: заключенному приходила ТАКАЯ сумма, он попадал под подозрение. ТАКОЙ
                   1661: предмет мебели упоминался в статье о противостоянии граждан и
                   1662: чиновников. Какое слово мы заменили на "ТАКОЙ"?
                   1663: 
                   1664: Ответ:
                   1665: Некруглый.
                   1666: 
                   1667: Комментарий:
                   1668: Оплата почтовых услуг производилась не дополнительно за счет
                   1669: отправителя, а из общей суммы денежного перевода. Государство не
                   1670: утруждало себя оплатой перевода, а частные лица обычно платили. Круглый
                   1671: стол предполагает равенство, а участники некруглого делятся на своих и
                   1672: чужих.
                   1673: 
                   1674: Источник:
                   1675:    1. А.И. Солженицын. В круге первом.
                   1676: http://www.flibusta.is/b/79959/read
                   1677:    2. http://www.mosoblpress.ru/regions/16/mass_media/3/89/item4835/
                   1678: 
                   1679: Автор:
                   1680: Игорь Демьянцев (Гомель)
                   1681: 
                   1682: Вопрос 11:
                   1683: Согласно акту о гербовом сборе, оформление документов предусматривало
                   1684: плату в пользу короля. В каком городе выступающая против этого акта
                   1685: толпа разгромила особняк вице-губернатора Хатчинсона?
                   1686: 
                   1687: Ответ:
                   1688: Бостон.
                   1689: 
                   1690: Комментарий:
                   1691: Жители Бостона, хоть и были подданными английского короля, но уже тогда
                   1692: были носителями антианглийских настроений, а вскоре даже устроили
                   1693: известное "чаепитие".
                   1694: 
                   1695: Источник:
                   1696:    1. http://ru.wikipedia.org/wiki/Акт_о_гербовом_сборе
                   1697:    2. http://en.wikipedia.org/wiki/Stamp_Act_1765
                   1698: 
                   1699: Автор:
                   1700: Александр Пономарёв (Пермь)
                   1701: 
                   1702: Вопрос 12:
                   1703: Хищник леоноптерикс из фильма "Аватар" предпочитает внезапно атаковать
                   1704: сверху. Его местное название означает "ТАКАЯ АЛЬФА", потому что ТАКОЙ
                   1705: жертва видит АЛЬФУ леоноптерикса. Какие слова мы заменили словами "ТАКАЯ
                   1706: АЛЬФА"?
                   1707: 
                   1708: Ответ:
                   1709: Последняя тень.
                   1710: 
                   1711: Комментарий:
                   1712: Последней жертва видит тень леоноптерикса.
                   1713: 
                   1714: Источник:
                   1715: http://hontor.ru/publ/ksenobiologija/filmy/russkaja_pandoropedija_leonopteryx_rex_velikij_leonopteriks/15-1-0-114
                   1716: 
                   1717: Автор:
                   1718: Максим Драчёв (Кемерово)
                   1719: 
                   1720: Вопрос 13:
                   1721: Выяснив мотив одного дела об угоне самолета, следствие скрыло имя
                   1722: преступника. По мнению следователя Анатолия Квашнина, одного ЕГО в
                   1723: истории человечества достаточно. Назовите ЕГО.
                   1724: 
                   1725: Ответ:
                   1726: Герострат.
                   1727: 
                   1728: Комментарий:
                   1729: Как и Герострат, сжегший храм Артемиды, этот угонщик тоже хотел
                   1730: прославиться.
                   1731: 
                   1732: Источник:
                   1733: http://www.vz.ru/society/2008/10/16/219856.html
                   1734: 
                   1735: Автор:
                   1736: Григорий Теплых (Пермь)
                   1737: 
                   1738: Вопрос 14:
                   1739: На вершине горы Сьерра-де-ла-Неблина сохранилось множество эндемичных
                   1740: видов флоры и фауны. С каким творением человеческих рук сравнивают эту
                   1741: гору?
                   1742: 
                   1743: Ответ:
                   1744: С [Ноевым] ковчегом.
                   1745: 
                   1746: Комментарий:
                   1747: Там тоже было много разных видов животных.
                   1748: 
                   1749: Источник:
                   1750: Энциклопедия для детей. Страны. Народы. Цивилизации. - М.: Аванта+,
                   1751: 1999.
                   1752: 
                   1753: Автор:
                   1754: Григорий Теплых (Пермь)
                   1755: 
                   1756: Вопрос 15:
                   1757: (pic: 20150818.jpg)
                   1758:    Какое имя носил тот, чье тело мы от вас скрыли на этой фотографии?
                   1759: 
                   1760: Ответ:
                   1761: Хатико.
                   1762: 
                   1763: Комментарий:
                   1764: Тело должно быть небольшим. Лица восточные. Кстати, весьма вероятно, что
                   1765: в форму одеты не военные или полицейские, а железнодорожники.
                   1766: 
                   1767: Источник:
                   1768: http://www.adme.ru/tvorchestvo-fotografy/45-fotografij-kotorye-izmenyat-vashe-predstavlenie-o-proshlom-696660/
                   1769: 
                   1770: Автор:
                   1771: Игорь Демьянцев (Гомель)
                   1772: 
                   1773: Вопрос 16:
                   1774: Один из международных биологических проектов называется "ОНА океана".
                   1775: Считается, что в XVIII веке математик Леонард Эйлер сумел обработать ЕЕ
                   1776: результаты в одиночку. Назовите ЕЕ двумя словами.
                   1777: 
                   1778: Ответ:
                   1779: Перепись населения.
                   1780: 
                   1781: Источник:
                   1782:    1. http://ru.wikipedia.org/wiki/Перепись_населения_Океана
                   1783:    2. http://www.math.rsu.ru/mexmat/kvm/MME/dsarch/euler.html
                   1784: 
                   1785: Автор:
                   1786: Григорий Теплых (Пермь)
                   1787: 
                   1788: Вопрос 17:
                   1789: Председатель общества советско-непальской дружбы Юрий Малышев преподнес
                   1790: королю необычный фотоснимок, сделанный достаточно далеко от территории
                   1791: Непала. Где был Малышев с третьего по одиннадцатое апреля 1984 года?
                   1792: 
                   1793: Ответ:
                   1794: В космосе.
                   1795: 
                   1796: Зачет:
                   1797: На орбите; синонимичные ответы.
                   1798: 
                   1799: Комментарий:
                   1800: Подарком был снимок Непала, сделанный из космоса. На Эверест он не
                   1801: восходил, да и снимок был сделан далеко от территории Непала.
                   1802: 
                   1803: Источник:
                   1804:    1. А.Х. Везиров. Моя дипслужба.
                   1805: http://lit.lib.ru/w/wezirow_a_h/dipzluzhba.shtml
                   1806:    2. http://ru.wikipedia.org/wiki/Малышев,_Юрий_Васильевич
                   1807: 
                   1808: Автор:
                   1809: Русиф Гусейнов (Баку)
                   1810: 
                   1811: Вопрос 18:
                   1812: Рассказывают, что композитор Джеремайя Кларк, решившись на самоубийство,
                   1813: захотел выбрать способ - повеситься или утопиться. Что произошло в
                   1814: процессе определения способа, если в результате Кларк застрелился?
                   1815: 
                   1816: Ответ:
                   1817: Монета упала на ребро.
                   1818: 
                   1819: Зачет:
                   1820: Синонимичные ответы.
                   1821: 
                   1822: Комментарий:
                   1823: Пришлось искать третий способ.
                   1824: 
                   1825: Источник:
                   1826: http://ru.wikipedia.org/wiki/Кларк,_Джеремайя
                   1827: 
                   1828: Автор:
                   1829: Игорь Демьянцев (Гомель)
                   1830: 
                   1831: Вопрос 19:
                   1832: Героиня компьютерной игры "Bulletstorm" [буллетсторм], рассказывая о
                   1833: системе пополнения боеприпасов, утверждает, что солдаты с лучшей
                   1834: статистикой получают патроны, а другие - не получают. Какого англичанина
                   1835: ее собеседник упоминает в ответ?
                   1836: 
                   1837: Ответ:
                   1838: [Чарльз] Дарвин.
                   1839: 
                   1840: Комментарий:
                   1841: Героиня намекает на теорию естественного отбора, согласно которой
                   1842: выживают более приспособленные особи.
                   1843: 
                   1844: Источник:
                   1845: "Bulletstorm", действие 4, глава 3.
                   1846: 
                   1847: Автор:
                   1848: Андрей Волков (Воскресенск)
                   1849: 
                   1850: Вопрос 20:
                   1851: У персонажа-домоседа из произведения Гая Юлия Орловского обе ОНИ красиво
                   1852: и величественно опускались на плечи. Назовите ИХ.
                   1853: 
                   1854: Ответ:
                   1855: Щеки.
                   1856: 
                   1857: Комментарий:
                   1858: Персонаж вел малоподвижный образ жизни.
                   1859: 
                   1860: Источник:
                   1861: Гай Юлий Орловский. Ричард Длинные Руки - князь.
                   1862: http://www.loveread.ec/read_book.php?id=10801&p=36
                   1863: 
                   1864: Автор:
                   1865: Григорий Теплых (Пермь)
                   1866: 
                   1867: Вопрос 21:
                   1868: Датский комический актер Якоб Хаугард обещал больше попутного ветра на
                   1869: велодорожках, лучшие прогнозы погоды и большее разнообразие подарков к
                   1870: Рождеству. Назовите его место работы с 1994 по 1998 год словом
                   1871: французского происхождения.
                   1872: 
                   1873: Ответ:
                   1874: Парламент.
                   1875: 
                   1876: Комментарий:
                   1877: Эти обещания входили в его предвыборную программу.
                   1878: 
                   1879: Источник:
                   1880:    1. http://en.wikipedia.org/wiki/Jacob_Haugaard
                   1881:    2. http://sciencelibrary.ru/datskiy-aktyor-komik-yakob-haugard-s-1979-goda-ballotirovalsya-v-parlament-svoey-stranyi-s-shutochnoy-programmoy/
                   1882:    3. М.Г. Вершовский. А другого глобуса у вас нет?..
                   1883: http://www.flibusta.net/b/73735/read
                   1884:    4. http://da.wikipedia.org/wiki/Folketinget
                   1885: 
                   1886: Автор:
                   1887: Игорь Демьянцев (Гомель)
                   1888: 
                   1889: Вопрос 22:
                   1890: Согласно так называемой "Новой аналитической энциклопедии", задолго до
                   1891: появления известного изобретения перед выходом из дома девушка надевала
                   1892: ИКС с написанным посланием. Назовите ИКС.
                   1893: 
                   1894: Ответ:
                   1895: Сарафан.
                   1896: 
                   1897: Комментарий:
                   1898: Так в этой несерьезной энциклопедии объясняется выражение "сарафанное
                   1899: радио".
                   1900: 
                   1901: Источник:
                   1902: http://ru-encyclopedia.livejournal.com/141775.html
                   1903: 
                   1904: Автор:
                   1905: Максим Драчёв (Кемерово)
                   1906: 
                   1907: Вопрос 23:
                   1908: (pic: 20150819.jpg)
                   1909:    Боб Дилан, исполняя популярную на Западе песню, вместо ИХ имен
                   1910: упомянул президентов США, указанных в раздаточном материале. Назовите ИХ
                   1911: двумя словами.
                   1912: 
                   1913: Ответ:
                   1914: Олени Санта-Клауса.
                   1915: 
                   1916: Зачет:
                   1917: Олени Санты.
                   1918: 
                   1919: Комментарий:
                   1920: На раздаточном материале фамилии президентов расположены подобно оленям
                   1921: в упряжке Санта-Клауса. У Боба Дилана есть своя кавер-версия песни "Must
                   1922: be Santa" [маст би сАнта]. Первоначально оленей у Санты было восемь,
                   1923: Рудольф добавился позже.
                   1924: 
                   1925: Источник:
                   1926:    1. http://genius.com/Bob-dylan-must-be-santa-lyrics
                   1927:    2. http://en.wikipedia.org/wiki/List_of_Presidents_of_the_United_States
                   1928: 
                   1929: Автор:
                   1930: Игорь Коршунов (Коломна), Андрей Волков (Воскресенск)
                   1931: 
                   1932: Вопрос 24:
                   1933: Президент Венгрии Арпад Гёнц участвовал в открытии заводов "Audi"
                   1934: [Ауди]. Кроме того, он перевел на венгерский язык книгу, которую автор
                   1935: начал писать в 1930-е годы. Какую именно книгу?
                   1936: 
                   1937: Ответ:
                   1938: "Властелин Колец".
                   1939: 
                   1940: Комментарий:
                   1941: На эмблеме автоконцерна "Audi" четыре кольца. Книга вышла довольно
                   1942: объемной, и Толкиен писал ее не один год.
                   1943: 
                   1944: Источник:
                   1945:    1. http://ru.wikipedia.org/wiki/Гёнц,_Арпад
                   1946:    2. http://en.wikipedia.org/wiki/The_Lord_of_the_Rings
                   1947: 
                   1948: Автор:
                   1949: Дмитрий Богданов (Белгород)
                   1950: 
                   1951: Вопрос 25:
                   1952: Одна студия дизайна предложила изобразить на страницах норвежского
                   1953: паспорта природные ландшафты. По задумке дизайнеров, просвечивание
                   1954: ультрафиолетом позволит полюбоваться ЭТИМ. На планетах-гигантах из-за
                   1955: сильного магнитного поля ЭТО имеет гораздо бОльшие масштабы. Назовите
                   1956: ЭТО.
                   1957: 
                   1958: Ответ:
                   1959: Северное сияние.
                   1960: 
                   1961: Зачет:
                   1962: Полярное сияние.
                   1963: 
                   1964: Источник:
                   1965:    1. http://nordicdesign.ru/peyzazh-v-pastelnyih-tonah-novyiy-dizayn-norvezhskogo-pasporta/
                   1966:    2. http://ru.wikipedia.org/wiki/Полярное_сияние
                   1967: 
                   1968: Автор:
                   1969: Игорь Демьянцев (Гомель)
                   1970: 
                   1971: Вопрос 26:
                   1972: В повести Сергея Валяева поэт сравнил луну с ИКСОМ в пасти ночи.
                   1973: Назовите ИКС двумя словами, начинающимися на одну и ту же букву.
                   1974: 
                   1975: Ответ:
                   1976: Золотой зуб.
                   1977: 
                   1978: Комментарий:
                   1979: Так же сверкает в окружении темноты.
                   1980: 
                   1981: Источник:
                   1982: С. Валяев. Качает чёрт качели. // "Юность", 1980, N 1.
                   1983: 
                   1984: Автор:
                   1985: Евгений Котович (Кишинев)
                   1986: 
                   1987: Вопрос 27:
                   1988: В аннотации к книге Алексея Шепелёва сказано, что сюжетные линии
                   1989: переплетаются, перетекают друг в друга, но не пересекаются. Далее
                   1990: композиция романа сравнивается с тем, что можно сделать, например, из
                   1991: бумаги. С чем именно?
                   1992: 
                   1993: Ответ:
                   1994: С лентой Мёбиуса.
                   1995: 
                   1996: Источник:
                   1997:    1. http://magazines.russ.ru/znamia/2004/10/uric25.html
                   1998:    2. http://ru.wikipedia.org/wiki/Лента_Мёбиуса
                   1999: 
                   2000: Автор:
                   2001: Александр Симаков (Гомель)
                   2002: 
                   2003: Вопрос 28:
                   2004: В книге "Порри Гаттер и Каменный Философ" ОН был настойчив и даже тряс
                   2005: Порри за плечо. От какого глагола произошло ЕГО название?
                   2006: 
                   2007: Ответ:
                   2008: Будить.
                   2009: 
                   2010: Комментарий:
                   2011: ОН - будильник, а в мире магии будильники тоже умеют многое.
                   2012: 
                   2013: Источник:
                   2014: А.В. Жвалевский, И.Е. Мытько. Порри Гаттер и Каменный Философ.
                   2015: http://www.flibusta.is/b/66650/read
                   2016: 
                   2017: Автор:
                   2018: Григорий Теплых (Пермь)
                   2019: 
                   2020: Вопрос 29:
                   2021: Жена художника Данте Габриэля Россетти умерла от передозировки. На
                   2022: портрете жены он изобразил, как на ее ладонь птица кладет ИКС. Правда,
                   2023: на картине ИКС вовсе не красный. Назовите ИКС коротким словом.
                   2024: 
                   2025: Ответ:
                   2026: Мак.
                   2027: 
                   2028: Источник:
                   2029: http://ru.wikipedia.org/wiki/Beata_Beatrix
                   2030: 
                   2031: Автор:
                   2032: Русиф Гусейнов (Баку)
                   2033: 
                   2034: Вопрос 30:
                   2035: Мария Спинали изобрела "умную" модель купальника. Для использования
                   2036: покупательница купальника указывает ЕЕ тип и ЕГО желаемый уровень.
                   2037: Назовите ЕГО и ЕЕ.
                   2038: 
                   2039: Ответ:
                   2040: Загар, кожа.
                   2041: 
                   2042: Комментарий:
                   2043: Бикини предупреждает носительницу о достижении нужной степени загара,
                   2044: исходя из типа кожи и засекаемого времени.
                   2045: 
                   2046: Источник:
                   2047: http://geektimes.ru/post/251950/
                   2048: 
                   2049: Автор:
                   2050: Игорь Демьянцев (Гомель)
                   2051: 
                   2052: Вопрос 31:
                   2053: Герой корейской сказки Пак поймал волшебного бурундука. Вскоре после
                   2054: этого Пак сильно похудел, ведь с ним произошло почти то же, что и с
                   2055: более известным персонажем. Какое имя носил этот персонаж?
                   2056: 
                   2057: Ответ:
                   2058: Мидас.
                   2059: 
                   2060: Комментарий:
                   2061: В награду за освобождение жадный Пак попросил у волшебного зверя, чтобы
                   2062: всё, к чему он прикасался, превращалось бы в серебро.
                   2063: 
                   2064: Источник:
                   2065:    1. http://skazki.engindoc.com/index.php?option=com_content&task=category&sectionid=4&id=30&Itemid=29
                   2066:    2. http://ru.wikipedia.org/wiki/Мидас
                   2067: 
                   2068: Автор:
                   2069: Евгений Котович (Кишинев)
                   2070: 
                   2071: Вопрос 32:
                   2072: (pic: 20150820.jpg)
                   2073:    Созданием чего знаменит человек, изображенный с женой на этом фото?
                   2074: 
                   2075: Ответ:
                   2076: Баскетбола.
                   2077: 
                   2078: Комментарий:
                   2079: Это - преподаватель физического воспитания Джеймс Нейсмит. У него в
                   2080: руках прототип баскетбольной корзины, у жены - мяч.
                   2081: 
                   2082: Источник:
                   2083: http://www.adme.ru/tvorchestvo-fotografy/malo-kto-videl-eti-redkie-istoricheskie-foto-890310/
                   2084: 
                   2085: Автор:
                   2086: Игорь Демьянцев (Гомель)
                   2087: 
                   2088: Вопрос 33:
                   2089: Марк Твен писал о находящихся на северо-востоке США гостиницах с
                   2090: воинственными названиями, заставляющими благодарить доброго портье,
                   2091: который, вместо того чтобы СДЕЛАТЬ ЭТО, отпустит путника восвояси. Что
                   2092: именно сделать?
                   2093: 
                   2094: Ответ:
                   2095: Снять скальп.
                   2096: 
                   2097: Комментарий:
                   2098: Гостиницам давали имена индейцев.
                   2099: 
                   2100: Источник:
                   2101: М. Твен. Позолоченный век. http://www.flibusta.is/b/286470/read
                   2102: 
                   2103: Автор:
                   2104: Игорь Демьянцев (Гомель)
                   2105: 
                   2106: Вопрос 34:
                   2107: В сериале, основанном на классическом сюжете, ИХ предводителя сыграл
                   2108: Джейсон Лондон. Кто ОНИ?
                   2109: 
                   2110: Ответ:
                   2111: Аргонавты.
                   2112: 
                   2113: Комментарий:
                   2114: Джейсон сыграл Ясона. В английском языке эти имена даже пишутся
                   2115: одинаково.
                   2116: 
                   2117: Источник:
                   2118: http://en.wikipedia.org/wiki/Jason_and_the_Argonauts_(TV_miniseries)
                   2119: 
                   2120: Автор:
                   2121: Русиф Гусейнов (Баку)
                   2122: 
                   2123: Вопрос 35:
                   2124: Среди ПЕРВЫХ были "Фаворит", "Сюрприз", "Золотая рыбка" и даже
                   2125: "Новоселье". Памятник ВТОРОЙ установлен на автовокзале Нью-Йорка. Когда
                   2126: в Ленинграде ужесточились репрессии, Анна Кулишер сказала, что ПЕРВУЮ
                   2127: заменила ВТОРАЯ. Назовите и ПЕРВУЮ, и ВТОРУЮ.
                   2128: 
                   2129: Ответ:
                   2130: Лотерея, очередь.
                   2131: 
                   2132: Комментарий:
                   2133: Названия лотерей намекали на удачу или возможный выигрыш. На автовокзале
                   2134: поставлен памятник очереди в автобус. По мнению Анны Кулишер, если
                   2135: раньше репрессировали выборочно, то после ужесточения очередь дойдет до
                   2136: каждого.
                   2137: 
                   2138: Источник:
                   2139:    1. https://www.nalog.ru/rn77/news/archive/3776179/
                   2140:    2. http://mir.afisha.ru/sights/gorodskoy-avtovokzal
                   2141:    3. С.М. Волков. История культуры Санкт-Петербурга.
                   2142: http://www.flibusta.is/b/259795/read
                   2143: 
                   2144: Автор:
                   2145: Максим Кухтин (Березники)
                   2146: 
                   2147: Вопрос 36:
                   2148: Пишут, что для снятия психологической нагрузки изобретатель Джефферсон
                   2149: Славик предложил включать ИКС вместе с фонариком в придуманные им
                   2150: аварийные комплекты для жильцов многоэтажных домов. Наконечник ИКСА
                   2151: состоит из пористого материала - например, из нейлона. Назовите ИКС.
                   2152: 
                   2153: Ответ:
                   2154: Фломастер.
                   2155: 
                   2156: Зачет:
                   2157: Маркер.
                   2158: 
                   2159: Комментарий:
                   2160: Речь идет о наборе для застрявших в лифте. По мнению Славика, снять
                   2161: психологическую нагрузку должно разрисовывание стен. Фонарик - на случай
                   2162: отключения света в лифте.
                   2163: 
                   2164: Источник:
                   2165:    1. http://www.outdoors.ru/round/74/r74-4.php
                   2166:    2. http://ru.wikipedia.org/wiki/Фломастер
                   2167: 
                   2168: Автор:
                   2169: Валерий Леонченко (Кишинев)
                   2170: 
                   2171: Тур:
                   2172: 4 тур
                   2173: 
                   2174: Дата:
                   2175: 12-Feb-2016
                   2176: 
                   2177: Редактор:
                   2178: Алексей Полевой (Гомель) и Денис Рыбачук (Брест)
                   2179: 
                   2180: Инфо:
                   2181: Редакторы пакета благодарят за помощь в работе над пакетом команды
                   2182: "Ультиматум" (Гомель), "Три обезьяны" (Брест), Алексея Богословского
                   2183: (Санкт-Петербург), Наиля Фарукшина (Навои), Николая Лёгенького
                   2184: (Амстердам) и Евгения Миротина (Минск).
                   2185: 
                   2186: Вопрос 1:
                   2187: Появление пены в ходе некого процесса было сочтено символичным и при
                   2188: крещении стали использовать... Что?
                   2189: 
                   2190: Ответ:
                   2191: Шампанское.
                   2192: 
                   2193: Зачет:
                   2194: Бутылку шампанского.
                   2195: 
                   2196: Комментарий:
                   2197: Считается, что впервые бутылку вина о борт корабля разбили в британском
                   2198: Чатеме в конце XVII века. Во Франции вскоре стали использовать
                   2199: шампанское: обильная пена, вырывающаяся из бутылки, олицетворяет морскую
                   2200: волну, бьющую в борт. Крещением корабля называется ритуал спуска на
                   2201: воду, в ходе которого и используется шампанское.
                   2202: 
                   2203: Источник:
                   2204:    1. http://www.vokrugsveta.ru/quiz/222697/
                   2205:    2. http://ru.wikipedia.org/wiki/Ритуал_спуска_корабля_на_воду
                   2206: 
                   2207: Автор:
                   2208: Алексей Полевой (Гомель)
                   2209: 
                   2210: Вопрос 2:
                   2211: ОН родился и вырос в Италии. В одной телепередаче ОН шел по рынку и
                   2212: рассказывал о том, как задумал знаменитое произведение. Назовите
                   2213: заглавного героя этого произведения.
                   2214: 
                   2215: Ответ:
                   2216: Чиполлино.
                   2217: 
                   2218: Комментарий:
                   2219: На овощном рынке Джанни Родари рассказывал о "Чиполлино". Слова "вырос в
                   2220: Италии" - подсказка.
                   2221: 
                   2222: Источник:
                   2223: Д/ф "Гении и злодеи уходящей эпохи. Джанни Родари".
                   2224: http://tvkultura.ru/video/show/brand_id/21985/episode_id/1236215/
                   2225: 
                   2226: Автор:
                   2227: Денис Рыбачук (Брест)
                   2228: 
                   2229: Вопрос 3:
                   2230: На одной карикатуре ЕГО подушка находится на ветвях дерева. В команде
                   2231: автора вопроса существует виртуальная награда "Золотой ОН" - игроку,
                   2232: давшему правильный ответ на 61-й секунде. Назовите ЕГО.
                   2233: 
                   2234: Ответ:
                   2235: Жираф.
                   2236: 
                   2237: Комментарий:
                   2238: Поздно доходит правильная версия до такого игрока, совсем как до жирафа.
                   2239: 
                   2240: Источник:
                   2241:    1. http://old-crocodile.livejournal.com/193702.html
                   2242:    2. Личный опыт автора вопроса.
                   2243: 
                   2244: Автор:
                   2245: Денис Рыбачук (Брест)
                   2246: 
                   2247: Вопрос 4:
                   2248: В передаче "Фантасты-предсказатели" обсуждается рациональное зерно в
                   2249: проекте космического лифта, который придумал Кларк. Какое английское имя
                   2250: при этом упоминается?
                   2251: 
                   2252: Ответ:
                   2253: Джек.
                   2254: 
                   2255: Комментарий:
                   2256: Идея лифта схожа с тем, что происходило в сказке "Джек и бобовое
                   2257: зернышко", а фраза "который придумал Кларк" может напомнить вам фразу
                   2258: "который построил Джек".
                   2259: 
                   2260: Источник:
                   2261: Документальный сериал "Фантасты-предсказатели", 4-я серия "Артур Кларк".
                   2262: 
                   2263: Автор:
                   2264: Денис Рыбачук (Брест)
                   2265: 
                   2266: Вопрос 5:
                   2267: Бедный персонаж Оруэлла, собираясь в гости, предпочитает использовать
                   2268: нитки, а не мазать чернилами АЛЬФЫ. Чья АЛЬФА является устойчивым
                   2269: выражением?
                   2270: 
                   2271: Ответ:
                   2272: Ахиллеса.
                   2273: 
                   2274: Зачет:
                   2275: Ахилла.
                   2276: 
                   2277: Комментарий:
                   2278: Персонаж беден, и носки у него не новые. Но опускаться до того, чтобы
                   2279: мазать чернилами пятки, он не хочет. Номер вопроса - небольшая
                   2280: подсказка.
                   2281: 
                   2282: Источник:
                   2283: Дж. Оруэлл. Да здравствует фикус. http://www.flibusta.is/b/452078/read
                   2284: 
                   2285: Автор:
                   2286: Алексей Полевой (Гомель)
                   2287: 
                   2288: Вопрос 6:
                   2289: Персонаж Альберто МЕндеса выдумывает и рассказывает полковнику, который
                   2290: собирается вынести ему смертный приговор, всё новые подробности о его
                   2291: погибшем сыне. С кем себя сравнивает этот персонаж?
                   2292: 
                   2293: Ответ:
                   2294: Шахерезада.
                   2295: 
                   2296: Зачет:
                   2297: Шехерезада.
                   2298: 
                   2299: Комментарий:
                   2300: Полковник, конечно, не царь Шахрияр, но от него зависит жизнь
                   2301: рассказчика. Благодаря новым воспоминаниям следствие над подсудимым
                   2302: затягивается, ведь отцу интересно узнать о сыне новое.
                   2303: 
                   2304: Источник:
                   2305: А. Мендес. Слепые подсолнухи. http://www.flibusta.is/b/283995/read
                   2306: 
                   2307: Автор:
                   2308: Алексей Полевой (Гомель)
                   2309: 
                   2310: Вопрос 7:
                   2311: В рассказе Натаниэля ГОторна в музее выставлены различные сказочные
                   2312: предметы. ОНИ оказались в музее из-за развития железнодорожного
                   2313: транспорта. Назовите ИХ.
                   2314: 
                   2315: Ответ:
                   2316: Семимильные сапоги.
                   2317: 
                   2318: Зачет:
                   2319: Сапоги-скороходы.
                   2320: 
                   2321: Комментарий:
                   2322: Поезд и так быстро едет, а скоростным сапогам место в музее.
                   2323: 
                   2324: Источник:
                   2325: Н. Готорн. Собранье знатока. http://www.flibusta.is/b/292510/read#t2
                   2326: 
                   2327: Автор:
                   2328: Денис Рыбачук (Брест)
                   2329: 
                   2330: Вопрос 8:
                   2331: Реконструкторы Алла и Сергей ПоликарпУк построили драккАр на четыре пары
                   2332: весел. Как его назвали?
                   2333: 
                   2334: Ответ:
                   2335: СлейпнИр.
                   2336: 
                   2337: Комментарий:
                   2338: Драккар - корабль викингов, в данном случае с восемью веслами. Слейпнир
                   2339: - восьминогий конь из скандинавской мифологии, чье имя переводится как
                   2340: "скользящий", т.е. и кораблю тоже вполне подходит.
                   2341: 
                   2342: Источник:
                   2343: http://www.kobrincity.by/news/item/7421-drakkar.html
                   2344: 
                   2345: Автор:
                   2346: Денис Рыбачук (Брест)
                   2347: 
                   2348: Вопрос 9:
                   2349: Когда известный человек вступил в тайное общество на одном
                   2350: средиземноморском острове, прозвучала АЛЬФА. Правовая система США не
                   2351: распространяет действие АЛЬФЫ на террористов. Какие два слова мы
                   2352: заменили на АЛЬФУ?
                   2353: 
                   2354: Ответ:
                   2355: Клятва Гиппократа.
                   2356: 
                   2357: Комментарий:
                   2358: При вступлении в тайное общество неофиты обычно произносят клятвы.
                   2359: Подобную клятву произнес и Гиппократ, вступая в одно из обществ на
                   2360: острове Кос, хотя ее текст, наверное, существенно отличался от того, что
                   2361: сегодня называется клятвой Гиппократа. В США оказывать медицинскую
                   2362: помощь террористам совсем не обязательно.
                   2363: 
                   2364: Источник:
                   2365:    1. Документальный сериал "Проект Энциклопедия" (2006-2008), серия
                   2366: "Гиппократ". http://rutracker.org/forum/viewtopic.php?t=2201499
                   2367:    2. http://ru.wikipedia.org/wiki/Клятва_Гиппократа
                   2368: 
                   2369: Автор:
                   2370: Денис Рыбачук (Брест)
                   2371: 
                   2372: Вопрос 10:
                   2373: Дуплет.
                   2374:    1. Когда корабль Джеймса Кука сел на мель, за выброшенными в море
                   2375: пушками последовала ОНА. Назовите ЕЕ двумя словами.
                   2376:    2. Уильям Бернстайн считает, что дикий хлОпок распространился по миру
                   2377: благодаря тому, что семена не боятся ЕЕ. Назовите ЕЕ двумя словами.
                   2378: 
                   2379: Ответ:
                   2380:    1. Пресная вода.
                   2381:    2. Морская вода.
                   2382: 
                   2383: Зачет:
                   2384:    1. Питьевая вода.
                   2385:    2. Соленая вода.
                   2386: 
                   2387: Комментарий:
                   2388:    1. В морскую воду было выброшено самое тяжелое и даже самое ценное.
                   2389: Бочки сохраняли в надежде набрать пресную воду на островах.
                   2390:    2. Семена хлопка распространялись по миру в том числе и посредством
                   2391: морских течений.
                   2392: 
                   2393: Источник:
                   2394:    1. Н.К. Чуковский. Водители фрегатов.
                   2395: http://www.flibusta.is/b/82191/read
                   2396:    2. У. Бернстайн. Великолепный обмен. История мировой торговли.
                   2397: http://coollib.com/b/272395/read
                   2398: 
                   2399: Автор:
                   2400: Денис Рыбачук (Брест)
                   2401: 
                   2402: Вопрос 11:
                   2403: В произведении Захара Прилепина ребенок, гостивший у бабушки, сравнивает
                   2404: ЭТО с немецким шлемом времен кайзеровской империи. Назовите ЭТО
                   2405: двухкоренным словом.
                   2406: 
                   2407: Ответ:
                   2408: Рукомойник.
                   2409: 
                   2410: Комментарий:
                   2411: Характерный деревенский рукомойник напоминает ребенку перевернутый
                   2412: кайзеровский немецкий шлем. Предыдущий вопрос про воду мог задать
                   2413: правильное направление обсуждения.
                   2414: 
                   2415: Источник:
                   2416: З. Прилепин. Грех. http://www.flibusta.is/b/143531/read
                   2417: 
                   2418: Автор:
                   2419: Денис Рыбачук (Брест)
                   2420: 
                   2421: Вопрос 12:
                   2422: Индейский вождь АтуЭй является кубинским героем. По легенде, Атуэй
                   2423: отказался ДЕЛАТЬ ЭТО, чтобы снова не оказаться среди белых людей. Что
                   2424: делать?
                   2425: 
                   2426: Ответ:
                   2427: Креститься.
                   2428: 
                   2429: Зачет:
                   2430: Принять христианство; перейти в христианство; стать христианином;
1.4     ! rubashki 2431: принять католичество/католицизм; перейти в католичество/католицизм;
1.1       rubashki 2432: стать католиком.
                   2433: 
                   2434: Комментарий:
                   2435: Герой боролся против конкистадоров и попал в плен. Перед казнью ему
                   2436: предложили креститься, чтобы попасть в рай, но Атуэй предпочел оказаться
                   2437: вместе со своими воинами в аду.
                   2438: 
                   2439: Источник:
                   2440: http://ru.wikipedia.org/wiki/Атуэй
                   2441: 
                   2442: Автор:
                   2443: Алексей Полевой (Гомель)
                   2444: 
                   2445: Вопрос 13:
                   2446: Вскоре после победы в Великой Отечественной войне, из Германии
                   2447: известному человеку был прислан подарок из фарфора. Один блогер считает,
                   2448: что подарок был ЕЮ. Назовите ЕЕ устойчивым выражением.
                   2449: 
                   2450: Ответ:
                   2451: Трубка мира.
                   2452: 
                   2453: Комментарий:
                   2454: Курительные трубки изготавливают из разных материалов, в том числе и из
                   2455: фарфора. Подарок вождю СССР - Иосифу Сталину - символичен ввиду
                   2456: индейского обычая "раскуривать трубку мира" после окончания войны.
                   2457: 
                   2458: Источник:
                   2459: http://nnm.me/blogs/konelav/kollekciya-trubok-stalina/
                   2460: 
                   2461: Автор:
                   2462: Алексей Полевой (Гомель)
                   2463: 
                   2464: Вопрос 14:
                   2465: На средневековых картинах на определенный сюжет ОН обычно изображался в
                   2466: стороне от основных персонажей. Так, на картине одного нижнерейнского
                   2467: художника ОН сидит в стороне с топором. Назовите ЕГО.
                   2468: 
                   2469: Ответ:
                   2470: [Святой] Иосиф.
                   2471: 
                   2472: Комментарий:
                   2473: Иосиф, как и положено ему, плотничает и не имеет никакого отношения к
                   2474: рождению Христа. Иосиф Сталин из предыдущего вопроса никакого отношения
                   2475: к этому вопросу не имеет.
                   2476: 
                   2477: Источник:
                   2478:    1. Я. Вег. Немецкая станковая живопись семнадцатого века.
                   2479:    2. http://www.liveinternet.ru/community/lj_npl_22/post339015129/
                   2480: 
                   2481: Автор:
                   2482: Денис Рыбачук (Брест)
                   2483: 
                   2484: Вопрос 15:
                   2485: Алексей Аракчеев происходил из обедневшего дворянского рода Новгородской
                   2486: губернии. Желая сделать карьеру, юный Аракчеев даже хотел повторить
                   2487: поступок... Кого?
                   2488: 
                   2489: Ответ:
                   2490: [Михайло] Ломоносова.
                   2491: 
                   2492: Комментарий:
                   2493: Юный Аракчеев тоже решил идти в Москву пешком с обозами.
                   2494: 
                   2495: Источник:
                   2496: С. Шубинский. Картины прошлого.
                   2497: http://rutracker.org/forum/viewtopic.php?t=1873275
                   2498: 
                   2499: Автор:
                   2500: Денис Рыбачук (Брест)
                   2501: 
                   2502: Вопрос 16:
                   2503: В программу зимних Олимпийских игр 1928 года входил скъёринг. Для
                   2504: занятия скъёрингом необходимы лошадь, ПЕРВЫЙ и ВТОРЫЕ. Назовите летний
                   2505: вид спорта, для которого также необходимы ПЕРВЫЙ и ВТОРЫЕ.
                   2506: 
                   2507: Ответ:
                   2508: Водные лыжи.
                   2509: 
                   2510: Зачет:
                   2511: Воднолыжный спорт.
                   2512: 
                   2513: Комментарий:
                   2514: Скъёринг - скандинавский вид спорта, куда там без лыж. :-) Лыжника тянет
                   2515: по трассе лошадь, при этом ездоку на ходу необходимо преодолевать
                   2516: препятствия. В водных лыжах и.о. лошади - катер, а ПЕРВЫЙ - трос (или
                   2517: буксировочный фал) - и ВТОРЫЕ - лыжи - остаются неизменными, только
                   2518: слегка меняют свою форму. Кстати, другое название скъёринга -
                   2519: скиджоринг, а буквы "ск" в его начале - отсылка к слову "ski" [ски] -
                   2520: лыжи.
                   2521: 
                   2522: Источник:
                   2523:    1. http://sport.tut.by/news/aboutsport/478688.html
                   2524:    2. http://ru.wikipedia.org/wiki/Скиджоринг
                   2525:    3. http://ru.wikipedia.org/wiki/Зимние_Олимпийские_игры_1928
                   2526: 
                   2527: Автор:
                   2528: Алексей Полевой (Гомель)
                   2529: 
                   2530: Вопрос 17:
                   2531: Персонаж одного готического фильма во время ужина понимает, что ЭТО
                   2532: поддельное. Назовите ЭТО двумя словами, начинающимися на одну букву.
                   2533: 
                   2534: Ответ:
                   2535: Столовое серебро.
                   2536: 
                   2537: Комментарий:
                   2538: Было бы серебро настоящим, персонаж сразу бы это почувствовал, ведь он -
                   2539: вампир.
                   2540: 
                   2541: Источник:
                   2542: Х/ф "Мрачные тени" (2012), реж. Тим Бёртон.
                   2543: 
                   2544: Автор:
                   2545: Денис Рыбачук (Брест)
                   2546: 
                   2547: Вопрос 18:
                   2548: В средневековье считалось, что производить эту процедуру надо с плотно
                   2549: закрытым ртом. Назовите ее словом греческого происхождения.
                   2550: 
                   2551: Ответ:
                   2552: Экзорцизм.
                   2553: 
                   2554: Комментарий:
                   2555: Иначе изгнанный дух может вселиться в проводящего ритуал.
                   2556: 
                   2557: Источник:
                   2558: Г. Хаггард. От знахаря до врача. История науки врачевания.
                   2559: http://www.loveread.ec/read_book.php?id=44533&p=71
                   2560: 
                   2561: Автор:
                   2562: Денис Рыбачук (Брест)
                   2563: 
                   2564: Вопрос 19:
                   2565: В XVII веке некое занятие было шумным и небезопасным. Жан-Батист ЛюллИ
                   2566: умер от раны в ногу, нанесенной самому себе наконечником трости.
                   2567: Ответьте словом французского происхождения, чем он занимался в момент
                   2568: ранения.
                   2569: 
                   2570: Ответ:
                   2571: Дирижировал.
                   2572: 
                   2573: Зачет:
                   2574: Дирижирование.
                   2575: 
                   2576: Комментарий:
                   2577: В XVII веке руководитель оркестра при дирижировании отбивал такт, ударяя
                   2578: тростью об пол, что создавало шум, мешая слушать музыку, а также было
                   2579: небезопасно.
                   2580: 
                   2581: Источник:
                   2582:    1. http://ru.wikipedia.org/wiki/Люлли,_Жан-Батист
                   2583:    2. http://ru.wikipedia.org/wiki/Дирижёр
                   2584: 
                   2585: Автор:
                   2586: Алексей Полевой (Гомель)
                   2587: 
                   2588: Вопрос 20:
                   2589: Жан-Батист ЛюллИ был музыкантом при дворе Людовика XIV. В зените славы
                   2590: ему было поручено управление театром Пале-Рояль, но через некоторое
                   2591: время он скончался. Назовите заглавного персонажа оперы Люлли, премьера
                   2592: которой состоялась в 1683 году.
                   2593: 
                   2594: Ответ:
                   2595: Фаэтон.
                   2596: 
                   2597: Комментарий:
                   2598: Король-Солнце доверил Люлли управлять французским театром, что,
                   2599: косвенно, привело к трагической кончине музыканта. Фаэтону также было
                   2600: дано ответственное поручение, приведшее к его гибели. Слова "в зените" -
                   2601: подсказка.
                   2602: 
                   2603: Источник:
                   2604: http://ru.wikipedia.org/wiki/Люлли,_Жан-Батист
                   2605: 
                   2606: Автор:
                   2607: Денис Рыбачук (Брест)
                   2608: 
                   2609: Вопрос 21:
                   2610: (pic: 20150821.jpg)
                   2611:    "Мигрировала [пропуск] На Сицилию Регистрировать Фамилию". Если вы
                   2612: добавите в пропуск ЕЕ, то получится мнемоническое правило. Напишите ЕЕ.
                   2613: 
                   2614: Ответ:
                   2615: Соль.
                   2616: 
                   2617: Комментарий:
                   2618: На раздатке мы попытались разместить слова таким образом, как
                   2619: обозначаемые ими ноты (ми, си, ре и фа) располагаются на нотном стане, а
                   2620: фраза нужна для запоминания нот, которые располагаются на линиях стана.
                   2621: Слово "добавить" - подсказка.
                   2622: 
                   2623: Источник:
                   2624: http://ru.wikipedia.org/wiki/Нота_(музыка)
                   2625: 
                   2626: Автор:
                   2627: Алексей Полевой (Гомель)
                   2628: 
                   2629: Вопрос 22:
                   2630: Слова "ТАКАЯ", "ЭДАКАЯ" и "ИКС" - замены.
                   2631:    В романе Питера Мейла подземелье французского имения предлагается
                   2632: условно разделить на две половины - ТАКУЮ и ЭДАКУЮ. ТАКАЯ и ЭДАКАЯ
                   2633: половины есть на ИКСЕ, который был принят во время правления князя Шарля
                   2634: III. Назовите ИКС двумя словами.
                   2635: 
                   2636: Ответ:
                   2637: Флаг Монако.
                   2638: 
                   2639: Комментарий:
                   2640: Подземелье - это винный погреб, который делят на часть с белым вином и с
                   2641: красным. На флаге Монако тоже есть красная и белая половины.
                   2642: 
                   2643: Источник:
                   2644: П. Мейл. Афера с вином. http://www.flibusta.is/b/343314/read
                   2645: 
                   2646: Автор:
                   2647: Денис Рыбачук (Брест)
                   2648: 
                   2649: Вопрос 23:
                   2650: Антонио Меуччи проводил опыты по обезболиванию зубов с помощью
                   2651: электричества. Однажды, подключив провода к пациенту, он вышел в другую
                   2652: комнату, произвел разряд и услышал крик. С кем через несколько лет
                   2653: судился изобретатель?
                   2654: 
                   2655: Ответ:
                   2656: С [Александром] Беллом.
                   2657: 
                   2658: Комментарий:
                   2659: Крик Меуччи услышал по проводу, впоследствии ему пришла идея создания
                   2660: телефона. Но Белл смог обойти Меуччи в получении патента.
                   2661: 
                   2662: Источник:
                   2663: Документальный сериал "Mysteries at the Museum", s05e05, 34-я минута.
                   2664: 
                   2665: Автор:
                   2666: Денис Рыбачук (Брест)
                   2667: 
                   2668: Вопрос 24:
                   2669: Опросив десяток людей в США, журналист Николай Козлович пришел к
                   2670: неутешительному выводу, что ОНА - это незнание жителями классики.
                   2671: Назовите ЕЕ двумя словами.
                   2672: 
                   2673: Ответ:
                   2674: Американская трагедия.
                   2675: 
                   2676: Комментарий:
                   2677: "Американская трагедия" - классическое произведение Теодора Драйзера,
1.2       rubashki 2678: которое, как оказалось, в Америке не так уж известно.
1.1       rubashki 2679: 
                   2680: Источник:
                   2681:    1. http://people.onliner.by/2015/11/13/neformat
                   2682:    2. http://ru.wikipedia.org/wiki/Американская_трагедия
                   2683: 
                   2684: Автор:
                   2685: Иван Сергиевич (Минск)
                   2686: 
                   2687: Вопрос 25:
1.2       rubashki 2688: В течение некоторого времени в одной стране в начале ЭТОГО появлялись
1.1       rubashki 2689: богиня ЛАкшми и число 786, означающее хвалу Аллаху. Причиной был
                   2690: национальный обычай поклониться Высшим силам, прежде чем начинать
                   2691: рассказ. Назовите ЭТО одним словом.
                   2692: 
                   2693: Ответ:
                   2694: Киносеанс.
                   2695: 
                   2696: Зачет:
                   2697: Кинофильм; фильм.
                   2698: 
                   2699: Комментарий:
                   2700: Традиция индийская. Такой ритуал выполнялся в начале киносеанса в Индии.
                   2701: 
                   2702: Источник:
                   2703: А. Адига. Белый Тигр. http://www.flibusta.is/b/190839/read
                   2704: 
                   2705: Автор:
                   2706: Алексей Полевой (Гомель)
                   2707: 
                   2708: Вопрос 26:
                   2709: По мнению историка Дэвида Рейнольдса, конституция Третьей Республики
                   2710: была направлена на то, чтобы предотвратить появление ЕГО. Назовите ЕГО
                   2711: двумя словами, начинающимися на одну букву.
                   2712: 
                   2713: Ответ:
                   2714: Новый Наполеон.
                   2715: 
                   2716: Комментарий:
                   2717: Третья Республика существовала во Франции. Создатели конституции
                   2718: старались не допустить узурпации власти.
                   2719: 
                   2720: Источник:
                   2721: Документальный сериал "Длинные тени Первой мировой войны" ("The Long
                   2722: Shadow"), 2-я серия. http://rutracker.org/forum/viewtopic.php?t=5053220
                   2723: 
                   2724: Автор:
                   2725: Денис Рыбачук (Брест)
                   2726: 
                   2727: Вопрос 27:
                   2728: Созданные по государственному заказу работы дизайнера Маргарет КАлверт
                   2729: понятны и информативны. На одной из работ Маргарет изобразила себя в
                   2730: школьном возрасте. Ответьте двумя словами: дизайн чего разрабатывала
                   2731: Калверт?
                   2732: 
                   2733: Ответ:
                   2734: Дорожные знаки.
                   2735: 
                   2736: Комментарий:
                   2737: Маргарет утверждает, что на знаке "Дети" нарисован ее автопортрет.
                   2738: 
                   2739: Источник:
                   2740: Документальный сериал "Гении дизайна" ("The Genius of Design"), 4-я
                   2741: серия "Пластиковый дизайн".
                   2742: http://rutracker.org/forum/viewtopic.php?t=3303645
                   2743: 
                   2744: Автор:
                   2745: Денис Рыбачук (Брест)
                   2746: 
                   2747: Вопрос 28:
                   2748: (pic: 20150822.jpg)
                   2749:    Ответьте двумя словами: что, по замыслу художника, изображено на этой
                   2750: картинке?
                   2751: 
                   2752: Ответ:
                   2753: Духовные скрепы.
                   2754: 
                   2755: Комментарий:
                   2756: Известное после фразы Путина выражение в оригинальном видении художника.
                   2757: 
                   2758: Источник:
                   2759: http://teh-nomad.livejournal.com/1927709.html
                   2760: 
                   2761: Автор:
                   2762: Алексей Полевой (Гомель)
                   2763: 
                   2764: Вопрос 29:
                   2765: Существует мнение, что человекообразные были не хищниками, а
                   2766: падальщиками, ведь они могли каменными орудиями разделывать туши
                   2767: животных с прочной кожей. На том же основании автор книги "Удивительная
                   2768: палеонтология" относит к падальщикам ИХ. Назовите ИХ.
                   2769: 
                   2770: Ответ:
                   2771: Саблезубые [тигры].
                   2772: 
                   2773: Комментарий:
                   2774: Их зубы выполняли роль каменных орудий именно при разделке туши. Для
                   2775: охоты такие зубы были не пригодны.
                   2776: 
                   2777: Источник:
                   2778: К.Ю. Еськов. Удивительная палеонтология. История Земли и жизни на ней.
                   2779: http://www.flibusta.is/b/373975/read
                   2780: 
                   2781: Автор:
                   2782: Денис Рыбачук (Брест)
                   2783: 
                   2784: Вопрос 30:
                   2785: В одном из произведений Курта Воннегута ОН принюхивается и ведет
                   2786: хоботом. Назовите ЕГО двумя словами на одну букву.
                   2787: 
                   2788: Ответ:
                   2789: Танк "Тигр".
                   2790: 
                   2791: Комментарий:
                   2792: Воннегут известен своим участием во Второй мировой войне и антивоенной
                   2793: прозой. "Тигр" - немецкий танк, а хобот - это его ствол.
                   2794: 
                   2795: Источник:
                   2796: К. Воннегут. Бойня номер пять, или Крестовый поход детей.
                   2797: http://www.flibusta.is/b/316026/read
                   2798: 
                   2799: Автор:
                   2800: Денис Рыбачук (Брест)
                   2801: 
                   2802: Вопрос 31:
                   2803: Эта история произошла на самом деле. В 1775 году, чтобы поправить свое
                   2804: финансовое положение, Рудольф продал часть имущества местного
                   2805: ландсграфа. Заслушавшись объяснениями по данному поводу, представители
                   2806: закона дали ему сбежать. Назовите фамилию Рудольфа.
                   2807: 
                   2808: Ответ:
                   2809: Распе.
                   2810: 
                   2811: Зачет:
                   2812: Распэ.
                   2813: 
                   2814: Комментарий:
                   2815: История абсолютно правдивая. Считается, что люди, пришедшие его
                   2816: арестовывать, были так поражены его даром рассказчика, что дали Распе
                   2817: возможность скрыться. Впоследствии Распе стал автором записок
                   2818: Мюнхгаузена.
                   2819: 
                   2820: Источник:
                   2821: http://ru.wikipedia.org/wiki/Распе,_Рудольф_Эрих
                   2822: 
                   2823: Автор:
                   2824: Алексей Полевой (Гомель)
                   2825: 
                   2826: Вопрос 32:
                   2827: На картине Лукаса ван ФАлькенборха "Вид на Антверпен с ШЕльдой"
                   2828: изображены река, упавшая дама и кавалер в красных панталонах,
                   2829: выделяющихся на бледном фоне. Какое причастие в названии картины мы
                   2830: пропустили?
                   2831: 
                   2832: Ответ:
                   2833: Замерзшей.
                   2834: 
                   2835: Зачет:
                   2836: Заледеневшей; обледеневшей.
                   2837: 
                   2838: Комментарий:
                   2839: Как и многие голландские художники, ван Фалькенборх рисовал картины
                   2840: быта. На картине изображено катание на коньках по льду реки.
                   2841: 
                   2842: Источник:
                   2843:    1. https://commons.wikimedia.org/wiki/File:Lucas_van_Valkenborch_-_View_of_Antwerp_with_the_Frozen_Schelde_-_WGA24261.jpg
                   2844:    2. http://www.ng.ru/nauka/2012-02-22/13_cold.html
                   2845: 
                   2846: Автор:
                   2847: Алексей Полевой (Гомель)
                   2848: 
                   2849: Вопрос 33:
                   2850: Дуплет.
                   2851:    1. В индийской мифологии богиня судьбы рисует на АЛЬФАХ. Назовите
                   2852: АЛЬФУ.
                   2853:    2. По доминиканским поверьям, в лесах живут девушки-сигуАпы, отыскать
                   2854: которых непросто. От людей сигуапы отличаются тем, что их БЕТЫ повернуты
                   2855: назад. Назовите БЕТУ.
                   2856: 
                   2857: Ответ:
                   2858:    1. Ладонь.
                   2859:    2. Ступня.
                   2860: 
                   2861: Комментарий:
                   2862:    1. Богиня рисует линии судьбы.
                   2863:    2. Выследить по следам сигуапу не просто, можно запутаться, откуда и
                   2864: куда она шла.
                   2865: 
                   2866: Источник:
                   2867:    1. Когда улыбается удача: Индийские сказки, легенды и народные
                   2868: рассказы. / Сост. и пер. с англ. М.В. Кудиновой и В.М. Кудинова. - М.:
                   2869: Издательская фирма "Восточная литература" РАН, "Школа-Пресс", 1995.
                   2870:    2. http://www.artravel-dominicana.ru/religion/people
                   2871:    3. http://en.wikipedia.org/wiki/Ciguapa
                   2872: 
                   2873: Автор:
                   2874: Денис Рыбачук (Брест), Алексей Полевой (Гомель)
                   2875: 
                   2876: Вопрос 34:
                   2877: Средневековый монах из произведения Питера Акройда держит в руках ЕЕ и
                   2878: говорит, что перед ним целое стадо. Назовите ЕЕ одним словом.
                   2879: 
                   2880: Ответ:
                   2881: Книга.
                   2882: 
                   2883: Зачет:
                   2884: Библия.
                   2885: 
                   2886: Комментарий:
                   2887: "На каждую страницу идет целая овечья шкура. Получается, здесь перед
                   2888: нами овечье стадо, да не одно".
                   2889: 
                   2890: Источник:
                   2891: П. Акройд. Кларкенвельские рассказы.
                   2892: http://www.flibusta.is/b/295407/read
                   2893: 
                   2894: Автор:
                   2895: Алексей Полевой (Гомель)
                   2896: 
                   2897: Вопрос 35:
                   2898: Нодар Думбадзе пишет, что когда хочется спать, самое тяжелое - это ОНИ.
                   2899: Назовите ИХ.
                   2900: 
                   2901: Ответ:
                   2902: Веки.
                   2903: 
                   2904: Комментарий:
                   2905: Тяжело удержать их, не закрыть, когда хочется спать.
                   2906: 
                   2907: Источник:
                   2908: Н.В. Думбадзе. Я, Бабушка, Илико и Илларион.
                   2909: http://www.flibusta.is/b/89274/read
                   2910: 
                   2911: Автор:
                   2912: Кира Полевая (Гомель)
                   2913: 
                   2914: Вопрос 36:
                   2915: В столице Азербайджана, на улице АсАфа ЗейналлЫ, есть ресторан, который
                   2916: предлагает блюда французской и азербайджанской кухни. Напишите его
                   2917: название, состоящее из двух слов.
                   2918: 
                   2919: Ответ:
                   2920: "Merci Baku".
                   2921: 
                   2922: Зачет:
                   2923: "Мерси Баку".
                   2924: 
                   2925: Комментарий:
                   2926: Одно слово в названии - французское, а второе - азербайджанское -
                   2927: название столицы. Спасибо всем тем, кто участвовал в этом туре
                   2928: Студенческой лиги Сибири, и до новых встреч!
                   2929: 
                   2930: Источник:
                   2931: http://www.zoon.az/baku/restaurants/merci_baku/
                   2932: 
                   2933: Автор:
                   2934: Алексей Полевой (Гомель)
                   2935: 
                   2936: Тур:
                   2937: 5 тур
                   2938: 
                   2939: Дата:
                   2940: 11-Mar-2016
                   2941: 
                   2942: Редактор:
                   2943: Александр Пономарёв (Пермь)
                   2944: 
                   2945: Инфо:
                   2946: При подготовке пакета вопросов ценные замечания дали: Виктор Абдураманов
                   2947: (Березники), Александр Барышников (Добрянка), Иван Бердников (Пермь),
                   2948: Виктор Дзекановский (Санкт-Петербург), Илья Иванов (Пермь), Тимур
                   2949: Кафиатуллин (Москва), Константин Костенко (Гремячинск - Пермь), Сергей
                   2950: Лобачёв (Нижний Новгород), Нина Логинова (Пермь), Дмитрий Ломов (Пермь),
                   2951: Александр Марков (Москва), Александр Митряков (Ижевск), Айрат Мухарлямов
                   2952: (Казань), Вадим Опутин (Губаха - Пермь), Никита Сапрыкин (Воронеж),
                   2953: Алена Тарасова (Санкт-Петербург - Лондон), Лия Филатьева (Пермь),
                   2954: команда "Шпроты Шрёдингера", команды Пермского центра "Муравейник".
                   2955: 
                   2956: Вопрос 1:
                   2957: По сообщению журнала "ЭКЗО", у живущих в Сахаре туарегов ОНА наступает,
                   2958: когда начинает таять масло. Назовите ЕЕ.
                   2959: 
                   2960: Ответ:
                   2961: Весна.
                   2962: 
                   2963: Комментарий:
                   2964: У нас весна тоже наступает, когда всё вокруг начинает таять. А в Африке
                   2965: значительно теплее.
                   2966: 
                   2967: Источник:
                   2968: "ЭКЗО", N5, 2004. - С. 44.
                   2969: 
                   2970: Автор:
                   2971: Татьяна Аудерская (Одесса)
                   2972: 
                   2973: Вопрос 2:
                   2974: Один интернет-пользователь так прокомментировал мультфильм "Алеша
                   2975: Попович и Тугарин Змей": "Кому очень понравилось - пальцы вверх". В этом
                   2976: комментарии мы немного изменили одно слово. Как оно выглядело
                   2977: изначально?
                   2978: 
                   2979: Ответ:
                   2980: Палицы.
                   2981: 
                   2982: Комментарий:
                   2983: Палицы вверх.
                   2984: 
                   2985: Источник:
                   2986: http://www.youtube.com/watch?v=8vPQKM5UOJU (комментарий пользователя
                   2987: Кактус Мерлина)
                   2988: 
                   2989: Автор:
                   2990: Очир Очиров (Новосибирск)
                   2991: 
                   2992: Вопрос 3:
                   2993: Ученые признали, что запутались в ИХ названиях. Александр Тюрин
                   2994: уподобляет ИМ большие государства, которые, в отличие от небольших
                   2995: соседей, не смогли приспособиться к новым условиям. Назовите ИХ.
                   2996: 
                   2997: Ответ:
                   2998: Динозавры.
                   2999: 
                   3000: Комментарий:
                   3001: Плохо сохранившиеся найденные останки нередко не идентифицировались со
                   3002: своим видом, и нашедшие их палеонтологи объявляли о новом виде.
                   3003: Динозавры вымерли, а небольшие млекопитающие смогли приспособиться и
                   3004: выжили.
                   3005: 
                   3006: Источник:
                   3007:    1. http://lenta.ru/news/2008/09/17/dino/
                   3008:    2. http://fan.lib.ru/t/tjurin_a_w/tyurin-war-peace-ivan.shtml
                   3009: 
                   3010: Автор:
                   3011: Александр Пономарёв (Пермь)
                   3012: 
                   3013: Вопрос 4:
                   3014: В юности Махатма Ганди не только сам предавался греху, ДЕЛАЯ ЭТО, но и
                   3015: считал этот грех необходимым для всех индийцев. Ответьте двумя
                   3016: четырехбуквенными словами, что такое "ДЕЛАТЬ ЭТО".
                   3017: 
                   3018: Ответ:
                   3019: Есть мясо.
                   3020: 
                   3021: Комментарий:
                   3022: В юности он вопреки религиозным запретам стал есть мясо, чтобы окрепнуть
                   3023: физически. Тогда он считал, что всем индусам надо начать есть мясо,
                   3024: чтобы победить англичан. В дальнейшем Ганди стал убежденным
                   3025: вегетарианцем.
                   3026: 
                   3027: Источник:
                   3028: М. Ганди. Моя жизнь. http://www.flibusta.is/b/231270/read
                   3029: 
                   3030: Автор:
                   3031: Илья Шапиро (Иерусалим)
                   3032: 
                   3033: Вопрос 5:
                   3034: Поездку в Стамбул для заключения мира Кутузов умудрился растянуть на три
                   3035: месяца. Дело в том, что он взял с собой два десятка графов на случай,
                   3036: если договор подписать не удастся. Какие пять букв мы убрали из текста
                   3037: этого вопроса?
                   3038: 
                   3039: Ответ:
                   3040: карто.
                   3041: 
                   3042: Комментарий:
                   3043: Кутузов взял с собой картографов. За время остановок они тщательно
                   3044: зарисовывали карты местности по пути к Стамбулу.
                   3045: 
                   3046: Источник:
                   3047: "Военная тайна", Рен-ТВ, 16.08.2014 г.
                   3048: 
                   3049: Автор:
                   3050: Александр Бучацкий (Самара)
                   3051: 
                   3052: Вопрос 6:
                   3053: (pic: 20150823.jpg)
                   3054:    Перед вами ОНА образца 1804 года. Назовите ЕЕ максимально точно.
                   3055: 
                   3056: Ответ:
                   3057: Бубновая шестерка.
                   3058: 
                   3059: Зачет:
                   3060: Шестерка бубён; шестерка бубей.
                   3061: 
                   3062: Источник:
                   3063: http://www.wopc.co.uk/transformation/index
                   3064: 
                   3065: Автор:
                   3066: Игорь Демьянцев (Гомель)
                   3067: 
                   3068: Вопрос 7:
                   3069: Дмитрий Быков говорит, что во время войны великие дела пришлось
                   3070: совершать ИМ. Поэтому не удивительно, что после войны были опубликованы
                   3071: книги Николая Носова. Назовите ИХ двумя словами, начинающимися на
                   3072: соседние буквы алфавита.
                   3073: 
                   3074: Ответ:
                   3075: Маленькие люди.
                   3076: 
                   3077: Комментарий:
                   3078: Героями книг тоже стали маленькие люди.
                   3079: 
                   3080: Источник:
                   3081:    1. Д. Быков. Чиполлино и русское освободительное движение.
                   3082: https://vk.com/video-2048479_171463225 (63-я минута)
                   3083:    2. http://ru.wikipedia.org/wiki/Маленький_человек
                   3084: 
                   3085: Автор:
                   3086: Евгений Шерстобитов (Тольятти)
                   3087: 
                   3088: Вопрос 8:
                   3089: Для прокладки каналов и возведения плотин Мухаммед Али-Паша предложил
                   3090: СДЕЛАТЬ ЭТО. Но в результате стали использовать каменоломни. Что именно
                   3091: предложил Мухаммед Али-Паша?
                   3092: 
                   3093: Ответ:
                   3094: Разобрать пирамиды.
                   3095: 
                   3096: Зачет:
                   3097: Разобрать Сфинкса; синонимичные ответы.
                   3098: 
                   3099: Комментарий:
                   3100: Мухаммед Али-Паша был правителем Египта. Он предлагал пустить пирамиды
                   3101: на строительный камень.
                   3102: 
                   3103: Источник:
                   3104: http://en.wikipedia.org/wiki/Louis_Maurice_Adolphe_Linant_de_Bellefonds
                   3105: 
                   3106: Автор:
                   3107: Игорь Демьянцев (Гомель)
                   3108: 
                   3109: Вопрос 9:
                   3110: Герой мультсериала "Царь горы" ДЕЛАЕТ ЭТО при помощи олимпийского огня.
                   3111: В ответ на упрек, что он оскверняет олимпийский дух, герой говорит:
                   3112: "Наоборот, я им проникаюсь". Что же он делал?
                   3113: 
                   3114: Ответ:
                   3115: Прикуривал.
                   3116: 
                   3117: Зачет:
                   3118: Зажигал сигарету; синонимичные ответы.
                   3119: 
                   3120: Источник:
                   3121: Мультсериал "Царь горы", s06e07.
                   3122: 
                   3123: Автор:
                   3124: Андрей Волков (Воскресенск)
                   3125: 
                   3126: Вопрос 10:
                   3127: В 1989 году вышли мемуары одного из последних представителей этой
                   3128: профессии в Великобритании. Рассказывая об одержимости своим ремеслом,
                   3129: автор пишет, что "совершенно СОШЕЛ С УМА". Мы не спрашиваем, какие два
                   3130: слова мы заменили словами "СОШЕЛ С УМА". Кем работал этот человек?
                   3131: 
                   3132: Ответ:
                   3133: Палачом.
                   3134: 
                   3135: Комментарий:
                   3136: Мы заменили слова "потерял голову".
                   3137: 
                   3138: Источник:
                   3139: http://www.peoples.ru/military/butcher/syd_dernley/
                   3140: 
                   3141: Автор:
                   3142: Михаил Басс (Ярцево - Могилев)
                   3143: 
                   3144: Вопрос 11:
                   3145: Террорист Севдет Рамадан Бесим задумал начинить сумку взрывчаткой,
                   3146: однако был нокаутирован. В какой стране всё это произошло?
                   3147: 
                   3148: Ответ:
                   3149: В Австралии.
                   3150: 
                   3151: Комментарий:
                   3152: Он хотел подложить бомбу в сумку кенгуру, однако не учел, что они ведут
                   3153: себя агрессивно, если кто-то пытается засунуть что-нибудь в сумку.
                   3154: 
                   3155: Источник:
                   3156: http://www.dailymail.co.uk/news/article-3420357/Australian-accused-planning-pack-kangaroo-bomb.html
                   3157: 
                   3158: Автор:
                   3159: Валерий Семёнов (Минск - Могилев)
                   3160: 
                   3161: Вопрос 12:
                   3162: Рассуждая об улучшении отношений России с Китаем, СангаджИ ТарбАев
                   3163: заявил, что две страны разделял ОН, но лишь сейчас стало понятно, что
                   3164: это ОНА. Назовите ЕГО и ЕЕ.
                   3165: 
                   3166: Ответ:
                   3167: Амур, любовь.
                   3168: 
                   3169: Комментарий:
                   3170: Как известно, Амур - римский бог любви.
                   3171: 
                   3172: Источник:
                   3173: КВН. Встреча выпускников - 2015.
                   3174: 
                   3175: Автор:
                   3176: Александр Кошельник (Екатеринбург)
                   3177: 
                   3178: Вопрос 13:
                   3179: Константин Рыжов пишет, что любимым местом Александра III была
                   3180: Беловежская пуща. Мария Хейккинен пишет, что любимым местом Александра
                   3181: III был так называемый Царский камень. Какие два разных слова мы
                   3182: пропустили в тексте вопроса?
                   3183: 
                   3184: Ответ:
                   3185: Охоты, рыбалки.
                   3186: 
                   3187: Источник:
                   3188:    1. http://www.hrono.ru/biograf/bio_a/alexand3ry.php
                   3189:    2. http://www.nexplorer.ru/print/news__12616.htm
                   3190: 
                   3191: Автор:
                   3192: Валерий Семёнов (Минск - Могилев)
                   3193: 
                   3194: Вопрос 14:
                   3195: По мнению журнала "Вокруг света", английская идиома "засунуть носок в
                   3196: рот", что означает "замолкнуть", берет начало в XIX веке, когда у ИКСА
                   3197: еще не было регулятора. Назовите ИКС.
                   3198: 
                   3199: Ответ:
                   3200: Граммофон.
                   3201: 
                   3202: Зачет:
                   3203: Патефон.
                   3204: 
                   3205: Комментарий:
                   3206: Первые модели граммофона, появившиеся в XIX века, не имели регулятора
                   3207: громкости, и для приглушения звука слушатели засовывали носок в трубу.
                   3208: 
                   3209: Источник:
                   3210: http://www.vokrugsveta.ru/article/244145/
                   3211: 
                   3212: Автор:
                   3213: Александр Кошельник (Екатеринбург)
                   3214: 
                   3215: Вопрос 15:
                   3216: На берегах Оби и Ишима было много захоронений. По словам Виктора
                   3217: Сарианиди, в Западной Сибири ОНА началась уже в конце XVIII века.
                   3218: Назовите ЕЕ.
                   3219: 
                   3220: Ответ:
                   3221: Золотая лихорадка.
                   3222: 
                   3223: Комментарий:
                   3224: В Сибири могильники активно раскапывались в поисках золота. Американская
                   3225: золотая лихорадка началась на берегах другой реки - Клондайка.
                   3226: 
                   3227: Источник:
                   3228: В.И. Сарианиди. Афганистан: сокровища безымянных царей.
                   3229: http://www.istmira.com/razlichnoe/afganistan-sokrovishha-bezymyannyx-carej/page/36/
                   3230: 
                   3231: Автор:
                   3232: Марат Фрайман (Могилев)
                   3233: 
                   3234: Вопрос 16:
                   3235: Из-за манеры проводить бОльшую часть гонки за спинами соперников лыжника
                   3236: Петтера Нортуга назвали ИКСОМ. Сайт novatour.ru [новатур точка ру]
                   3237: указывает оптимальный объем мужского и женского ИКСОВ. Назовите ИКС.
                   3238: 
                   3239: Ответ:
                   3240: Рюкзак.
                   3241: 
                   3242: Комментарий:
                   3243: Сайт туристический, речь идет о размерах рюкзаков для походов: мужской -
                   3244: 80-90 литров, женский - 60-70 литров.
                   3245: 
                   3246: Источник:
                   3247:    1. http://www.skisport.ru/news/cross-country/71427/
                   3248:    2. http://www.novatour.ru/help_tyr/Beri-ryukzak-poshli-v-pokhod
                   3249: 
                   3250: Автор:
                   3251: Игорь Демьянцев (Гомель)
                   3252: 
                   3253: Вопрос 17:
                   3254: По рассказам одного репетитора, ОНА напоминает королеву, которая не
                   3255: выходит без свиты. Напишите ЕЕ.
                   3256: 
                   3257: Ответ:
                   3258: Q.
                   3259: 
                   3260: Комментарий:
                   3261: В словах, где употребляется эта буква, за ней почти всегда следует буква
                   3262: "U" [ю]. Слово "queen" [куИн] - "королева" - начинается с сочетания этих
                   3263: букв. Репетитором по английскому языку был автор этого вопроса.
                   3264: 
                   3265: Источник:
                   3266:    1. ЛНА.
                   3267:    2. http://en.wikipedia.org/wiki/Q
                   3268: 
                   3269: Автор:
                   3270: Дмитрий Богданов (Белгород)
                   3271: 
                   3272: Вопрос 18:
                   3273: Пишут, что за период с 1991 по 2004 год этот россиянин постарел всего на
                   3274: три года. По данным "Абсурдопедии", у него одна грудь и одно яичко. Кто
                   3275: он?
                   3276: 
                   3277: Ответ:
                   3278: Среднестатистический россиянин.
                   3279: 
                   3280: Зачет:
                   3281: Средний россиянин; синонимичные ответы.
                   3282: 
                   3283: Комментарий:
                   3284: Рождаемость тогда существенно снизилась, и за это время средний возраст
                   3285: жителей России увеличился на три года.
                   3286: 
                   3287: Источник:
                   3288:    1. http://www.rg.ru/2004/02/13/demografija.html
                   3289:    2. http://absurdopedia.wikia.com/wiki/Узкие_Круги
                   3290: 
                   3291: Автор:
                   3292: Александр Пономарёв (Пермь)
                   3293: 
                   3294: Вопрос 19:
                   3295: (pic: 20150824.jpg)
                   3296:    Диаграмма боковых и продольных сил, действующих на шину в каждый
                   3297: определенный момент времени, названа именем инженера Вунибальда Камма.
                   3298: Какая компания использовала его идеи в работе над изделием под номером
                   3299: 328?
                   3300: 
                   3301: Ответ:
                   3302: BMW.
                   3303: 
                   3304: Комментарий:
                   3305: Диаграмма напоминает контуры логотипа компании. На шасси BMW 328 было
                   3306: поставлено так называемое Камм-купэ.
                   3307: 
                   3308: Источник:
                   3309:    1. http://www.k2x2.info/hobbi_i_remesla/yekstremalnyi_avtotrening/p3.php
                   3310:    2. http://ru.wikipedia.org/wiki/Камм,_Вунибальд
                   3311:    3. http://en.wikipedia.org/wiki/Wunibald_Kamm
                   3312: 
                   3313: Автор:
                   3314: Ульяна Булавинцева (Москва)
                   3315: 
                   3316: Вопрос 20:
                   3317: В зимних путевых заметках Николай Пржевальский описывал привал
                   3318: экспедиции. Сначала на костре разогревался чай, готовился ужин, после
                   3319: чего подкрепившийся и отдохнувший автор начинал разогревать ИХ на
                   3320: костре. Назовите ИХ.
                   3321: 
                   3322: Ответ:
                   3323: Чернила.
                   3324: 
                   3325: Комментарий:
                   3326: Которыми он и писал свои зимние заметки.
                   3327: 
                   3328: Источник:
                   3329: Н.М. Пржевальский. Путешествие в Уссурийском крае. 1867-1869 гг.
                   3330: http://www.flibusta.is/b/353651/read
                   3331: 
                   3332: Автор:
                   3333: Валерий Семёнов (Минск - Могилев)
                   3334: 
                   3335: Вопрос 21:
                   3336: Салерно стали называть ИКСОМ после того, как там открылась медицинская
                   3337: школа. На самом деле ИКСОМ по праву можно назвать, например, Лариссу,
                   3338: центр греческой области Фессалия. Назовите ИКС двумя словами,
                   3339: начинающимися на одну и ту же букву.
                   3340: 
                   3341: Ответ:
                   3342: Город Гиппократа.
                   3343: 
                   3344: Комментарий:
                   3345: Знаменитый греческий врач Гиппократ прожил последние годы в Лариссе.
                   3346: 
                   3347: Источник:
                   3348:    1. http://www.biancoloto.com/salerno/salerno.html
                   3349:    2. http://ru.wikipedia.org/wiki/Гиппократ
                   3350: 
                   3351: Автор:
                   3352: Валерий Семёнов (Минск - Могилев)
                   3353: 
                   3354: Вопрос 22:
                   3355: Слоган обуви марки "Тотта" звучит так: "САМЫЙ размер, САМАЯ обувь".
                   3356: Восстановите два слова, замененные нами в этом слогане.
                   3357: 
                   3358: Ответ:
                   3359: Тот, та.
                   3360: 
                   3361: Комментарий:
                   3362: Название марки составлено из пропущенных слов.
                   3363: 
                   3364: Источник:
                   3365: http://www.parapyatok.ru/poleznaya-informaciya-o-detskoy-obuvi/totta-detskaya-obuv-tot-razmer-ta-obuv-totto.html
                   3366: 
                   3367: Автор:
                   3368: Александр Пономарёв (Пермь)
                   3369: 
                   3370: Вопрос 23:
                   3371: Говорят, что Папа Римский Лев I вышел из города, чтобы встретиться с
                   3372: королем, и даже убедил его воздержаться от излишних разрушений. Каким
                   3373: народом правил этот король?
                   3374: 
                   3375: Ответ:
                   3376: Вандалы.
                   3377: 
                   3378: Комментарий:
                   3379: Папа убедил короля вандалов Гейзериха воздержаться от излишних
                   3380: разрушений во время разграбления Рима. Тем не менее, разрушителей
                   3381: культурных ценностей стали называть вандалами.
                   3382: 
                   3383: Источник:
                   3384: http://ru.wikipedia.org/wiki/Разграбление_Рима_(455)
                   3385: 
                   3386: Автор:
                   3387: Александр Пономарёв (Пермь)
                   3388: 
                   3389: Вопрос 24:
                   3390: Герой Гришковца рассказывал, как старшие товарищи запрещали ему ДЕЛАТЬ
                   3391: ЭТО, но он чувствовал, что им было приятно, когда он ДЕЛАЛ ЭТО, и что им
                   3392: приятно запрещать. Что именно делать?
                   3393: 
                   3394: Ответ:
                   3395: Обращаться на "вы".
                   3396: 
                   3397: Источник:
                   3398: Е.В. Гришковец. Реки. http://www.flibusta.is/b/183416/read
                   3399: 
                   3400: Автор:
                   3401: Александр Пономарёв (Пермь)
                   3402: 
                   3403: Вопрос 25:
                   3404: Персонаж Марка Твена хвастает, что, когда он хочет порезвиться, он
                   3405: сплетает ПЕРВЫЕ и ВТОРЫЕ и ловит китов в Атлантическом океане. Назовите
                   3406: ПЕРВЫЕ и ВТОРЫЕ.
                   3407: 
                   3408: Ответ:
                   3409: Меридианы и параллели.
                   3410: 
                   3411: Зачет:
                   3412: В любом порядке.
                   3413: 
                   3414: Источник:
                   3415: М. Твен. Картинки прошлого. http://www.flibusta.is/b/57035/read
                   3416: 
                   3417: Автор:
                   3418: Игорь Демьянцев (Гомель)
                   3419: 
                   3420: Вопрос 26:
                   3421: В фильме "Дикая" совет опытного туриста САЖАТЬ ФИГИ после использования
                   3422: приводит к упоминанию фашистов. Какие два слова мы заменили словами
                   3423: "САЖАТЬ ФИГИ"?
                   3424: 
                   3425: Ответ:
                   3426: Сжигать книги.
                   3427: 
                   3428: Комментарий:
                   3429: Как известно, пришедший к власти Гитлер публично сжигал прогрессивные
                   3430: книги. Замена по созвучию.
                   3431: 
                   3432: Источник:
                   3433: Х/ф "Дикая" (2014), реж. Жан-Марк Валле, 47-я минута.
                   3434: 
                   3435: Автор:
                   3436: Ульяна Булавинцева (Москва)
                   3437: 
                   3438: Вопрос 27:
                   3439: (pic: 20150825.jpg)
                   3440:    Растение платицериум имеет второе название "ИКС". В Финляндии ИКСЫ
                   3441: иногда опрыскивают светоотражающим спреем. Назовите ИКСЫ двумя словами.
                   3442: 
                   3443: Ответ:
                   3444: Оленьи рога.
                   3445: 
                   3446: Зачет:
                   3447: Лосиные рога; рога оленя; рога лося.
                   3448: 
                   3449: Комментарий:
                   3450: Чтобы их обладатели не попадали под колеса.
                   3451: 
                   3452: Источник:
                   3453:    1. http://ru.wikipedia.org/wiki/Платицериум
                   3454:    2. http://www.ntv.ru/novosti/858860/
                   3455: 
                   3456: Автор:
                   3457: Андрей Ефремов (Могилев)
                   3458: 
                   3459: Вопрос 28:
                   3460: Персонаж романа Набокова "Смотри на арлекинов" носит фамилию с
                   3461: аристократической приставкой. Вероятно, он немного изменил
                   3462: первоначальную фамилию, которую носили и те, у кого на гербе был молот.
                   3463: От какого русского имени произошла первоначальная фамилия?
                   3464: 
                   3465: Ответ:
                   3466: Демид.
                   3467: 
                   3468: Комментарий:
                   3469: Персонаж носил фамилию де Мидов. Молот был на гербе известных заводчиков
                   3470: Демидовых.
                   3471: 
                   3472: Источник:
                   3473:    1. В.В. Набоков. Смотри на арлекинов.
                   3474: http://www.flibusta.is/b/159063/read
                   3475:    2. http://ru.wikipedia.org/wiki/Фамильная_приставка
                   3476:    3. http://ru.wikipedia.org/wiki/Демидовы
                   3477: 
                   3478: Автор:
                   3479: Александр Сидоренков (Смоленск)
                   3480: 
                   3481: Вопрос 29:
                   3482: Блогеру Сергею Анашкевичу не чужды сравнения с элементами мира природы.
                   3483: С чем он сравнил выглядывающие друг из-под друга таблички на домах?
                   3484: 
                   3485: Ответ:
                   3486: С годичными кольцами.
                   3487: 
                   3488: Зачет:
                   3489: С годовыми/древесными/дендрохронологическими кольцами.
                   3490: 
                   3491: Комментарий:
                   3492: Выглядывающие таблички советской эпохи словно напоминают о прошлом.
                   3493: 
                   3494: Источник:
                   3495: http://aquatek-filips.livejournal.com/985520.html
                   3496: 
                   3497: Автор:
                   3498: Александр Кошельник (Екатеринбург)
                   3499: 
                   3500: Вопрос 30:
                   3501: Говоря о слезах агента похоронной компании, Эрих Мария Ремарк упоминает
                   3502: искусственный ИКС. Слезы могут появиться по той же причине, что и ИКС.
                   3503: Назовите ИКС.
                   3504: 
                   3505: Ответ:
                   3506: Жемчуг.
                   3507: 
                   3508: Комментарий:
                   3509: Песчинка может вызвать как слезы, так и появление жемчужины.
                   3510: 
                   3511: Источник:
                   3512:    1. Э.М. Ремарк. Черный обелиск. http://www.flibusta.is/b/326124/read
                   3513:    2. http://ru.wikipedia.org/wiki/Жемчуг
                   3514: 
                   3515: Автор:
                   3516: Александр Сидоренков (Смоленск)
                   3517: 
                   3518: Вопрос 31:
                   3519: В фильме "Смертельное оружие" бандиты избивают пожилого героя, а затем
                   3520: ВОРОШАТ ПРОШЛОЕ, чтобы получить информацию. Какие четыре слова мы
                   3521: заменили словами "ВОРОШАТ ПРОШЛОЕ"?
                   3522: 
                   3523: Ответ:
                   3524: Сыплют соль на рану.
                   3525: 
                   3526: Источник:
                   3527: Х/ф "Смертельное оружие" (1987), реж. Ричард Доннер, 87-я минута.
                   3528: 
                   3529: Автор:
                   3530: Дмитрий Богданов (Белгород)
                   3531: 
                   3532: Вопрос 32:
                   3533: То ли из-за страха быть отравленным, то ли ради стремительности в битве
                   3534: Чингисхан употреблял ЕЕ. Назовите ЕЕ двумя словами, начинающимися на
                   3535: одну и ту же букву.
                   3536: 
                   3537: Ответ:
                   3538: Кровь коня.
                   3539: 
                   3540: Зачет:
                   3541: Конская кровь.
                   3542: 
                   3543: Комментарий:
                   3544: Не доверял предлагаемым напиткам и пил кровь прямо из вскрытой вены
                   3545: коня. Затем заклеивал дырку и скакал дальше.
                   3546: 
                   3547: Источник:
                   3548: В.В. Печорин. Бог и человек. Парадоксы откровения.
                   3549: http://www.flibusta.is/b/329782/read
                   3550: 
                   3551: Автор:
                   3552: Игорь Демьянцев (Гомель)
                   3553: 
                   3554: Вопрос 33:
                   3555: На одной учебной картинке на НЕЕ, облокотившись, прилег бегемот.
                   3556: Назовите ЕЕ словом греческого происхождения.
                   3557: 
                   3558: Ответ:
                   3559: Гипотенуза.
                   3560: 
                   3561: Комментарий:
                   3562: На картинке обыгрывается сходство слов "гипотенуза" и "гиппопотам".
                   3563: 
                   3564: Источник:
                   3565: (pic: 20150826.jpg)
                   3566: 
                   3567: Автор:
                   3568: Андрей Сметанин, Алексей Гилёв (Пермь)
                   3569: 
                   3570: Вопрос 34:
                   3571: В апартаментах одной пары ИХ была специальная телефонная будка, которая
                   3572: обеспечивала частичную конфиденциальность внутри пары. При этом не
                   3573: обращать внимания на происходящее ИХ учил иллюзионист Гарри Гудини.
                   3574: Назовите ИХ двумя словами.
                   3575: 
                   3576: Ответ:
                   3577: Сиамские близнецы.
                   3578: 
                   3579: Комментарий:
                   3580: Когда одна из сросшихся бедрами сестер Хилтон хотела пообщаться с
                   3581: кем-либо наедине, она закрывалась в будке, а другая в этот момент сидела
                   3582: по другую сторону двери и красила ногти.
                   3583: 
                   3584: Источник:
                   3585: http://paranormal-news.ru/news/zhizn_siamskikh_bliznecov_dejzi_i_violetty_khilton/2015-08-09-11160
                   3586: 
                   3587: Автор:
                   3588: Игорь Демьянцев (Гомель)
                   3589: 
                   3590: Вопрос 35:
                   3591: В одной компьютерной игре пират говорит, что понимает только язык АЛЬФЫ
                   3592: и язык ЭТОГО. В Российской империи награждали АЛЬФОЙ из ЭТОГО. Какие
                   3593: слова мы заменили словами "АЛЬФА" и "ЭТО"?
                   3594: 
                   3595: Ответ:
                   3596: Шпага, золото.
                   3597: 
                   3598: Зачет:
                   3599: Сабля, золото; оружие, золото.
                   3600: 
                   3601: Источник:
                   3602:    1. Компьютерная игра "Именем короля. Выборы".
                   3603:    2. http://ru.wikipedia.org/wiki/Золотое_оружие_%C2%ABЗа_храбрость%C2%BB
                   3604: 
                   3605: Автор:
                   3606: Андрей Ефремов (Минск - Могилев)
                   3607: 
                   3608: Вопрос 36:
                   3609: Согласно афоризму Станислава Ежи Леца, мы распяты на НИХ. Известная ИХ
                   3610: четверка сложена из кусочков бирмингемского опала. Назовите ИХ словом
                   3611: немецкого происхождения.
                   3612: 
                   3613: Ответ:
                   3614: Циферблаты.
                   3615: 
                   3616: Комментарий:
                   3617: Из бирмингемского опала сделаны циферблаты Биг Бена.
                   3618: 
                   3619: Источник:
                   3620:    1. http://citaty.info/quote/353160
                   3621:    2. http://ria.ru/spravka/20120626/685553543.html
                   3622: 
                   3623: Автор:
                   3624: Александр Бучацкий (Самара)
                   3625: 
                   3626: Тур:
                   3627: 6 тур
                   3628: 
                   3629: Дата:
                   3630: 25-Mar-2016
                   3631: 
                   3632: Редактор:
                   3633: Сергей Терентьев (Санкт-Петербург)
                   3634: 
                   3635: Инфо:
                   3636: Редактор благодарит за тестирование вопросов и ценные замечания Антона
                   3637: Волосатова, Александра Колышкина, Сергея Лобачёва, Максима Мерзлякова,
                   3638: Дмитрия Сахарова, Николая Слюняева, Наиля Фарукшина, Руслана Хаиткулова.
                   3639: 
                   3640: Вопрос 1:
                   3641: Забавно, но в статье о метрополитене говорится, что персонаж одной серии
                   3642: произведений хотел стать ИКСОМ, поэтому ему и прищемило хвост. Назовите
                   3643: ИКСА.
                   3644: 
                   3645: Ответ:
                   3646: Заяц.
                   3647: 
                   3648: Зачет:
                   3649: Зайчик.
                   3650: 
                   3651: Комментарий:
                   3652: До появления крутящихся турникетов в метро использовались турникеты,
                   3653: которые захлопывались при попытке неоплаченного прохода. Волк из "Ну,
                   3654: погоди!" хотел пройти в метро без билета, т.е. стать "зайцем".
                   3655: 
                   3656: Источник:
                   3657: http://www.fiesta.city/projects/itogi-goda-2015/muzey-metropolitena/
                   3658: 
                   3659: Автор:
                   3660: Сергей Терентьев (Санкт-Петербург)
                   3661: 
                   3662: Вопрос 2:
                   3663: Во время побега Кропоткина из тюрьмы сигналом от сообщника в
                   3664: лакированной шляпе должен был послужить ОН. Назовите ЕГО двумя словами.
                   3665: 
                   3666: Ответ:
                   3667: Солнечный зайчик.
                   3668: 
                   3669: Зачет:
                   3670: Солнечный заяц.
                   3671: 
                   3672: Комментарий:
                   3673: Соучастник побега должен был дать условный сигнал солнечным зайчиком при
                   3674: помощи своей блестящей шляпы.
                   3675: 
                   3676: Источник:
                   3677: http://www.vokrugsveta.ru/article/230279/
                   3678: 
                   3679: Автор:
                   3680: Сергей Терентьев (Санкт-Петербург)
                   3681: 
                   3682: Вопрос 3:
                   3683: Рассказывают, что для театральной постановки одному режиссеру
1.3       rubashki 3684: понадобился ИКС, и для реалистичности он проделал дыру в крыше. ИКС
1.1       rubashki 3685: входит в русскоязычное название сериала. Назовите ИКС двумя словами.
                   3686: 
                   3687: Ответ:
                   3688: Лунный свет.
                   3689: 
                   3690: Комментарий:
                   3691: Реалистичный лунный свет не получался, и режиссер проделал дыру в крыше.
                   3692: Этим режиссером, к слову, был Владимир Ворошилов. Сериал - "Детективное
                   3693: агентство "Лунный свет"".
                   3694: 
                   3695: Источник:
                   3696:    1. http://well-groomed.ru/stars/960-vladimir-voroshilov-biografiya-golos-za-kadrom.html
                   3697:    2. http://ru.wikipedia.org/wiki/Детективное_агентство_%C2%ABЛунный_свет%C2%BB
                   3698: 
                   3699: Автор:
                   3700: Сергей Терентьев (Санкт-Петербург)
                   3701: 
                   3702: Вопрос 4:
                   3703: Директор музея в городе СУндсвалль попросил своего знакомого воплотить в
                   3704: жизнь одну из баек некоего ХАкона ДАлмарка. Назовите профессию этого
                   3705: знакомого словом с двумя греческими корнями.
                   3706: 
                   3707: Ответ:
                   3708: Таксидермист.
                   3709: 
                   3710: Комментарий:
                   3711: (pic: 20150827.jpg)
                   3712:    Далмарк любил рассказывать охотничьи байки. Согласно одной из них, он
                   3713: увидел животное, передняя часть которого была заячьей, а задняя - от
                   3714: глухаря. Это животное, которое назвали "сквАдер", в шутку воплотил один
                   3715: таксидермист, сделав его чучело.
                   3716: 
                   3717: Источник:
                   3718:    1. http://www.bestiary.us/skvader
                   3719:    2. http://ru.wikipedia.org/wiki/Сквадер
                   3720:    3. http://ru.wikipedia.org/wiki/Таксидермия
                   3721: 
                   3722: Автор:
                   3723: Сергей Терентьев (Санкт-Петербург)
                   3724: 
                   3725: Вопрос 5:
                   3726: Согласно шутке, одна женщина так небрежно готовит, что вместо таймера
                   3727: использует ЕГО. Назовите ЕГО двумя словами, начинающимися на одну и ту
                   3728: же букву.
                   3729: 
                   3730: Ответ:
                   3731: Детектор дыма.
                   3732: 
                   3733: Зачет:
                   3734: Датчик дыма; дымовой детектор; дымовой датчик.
                   3735: 
                   3736: Комментарий:
                   3737: Женщина забывает о поставленной в духовке пище и не вспоминает о ней,
                   3738: пока дым не дойдет до детектора дыма и не сработает пожарная
                   3739: сигнализация.
                   3740: 
                   3741: Источник:
                   3742: http://anecdots.su/anecdot/137437
                   3743: 
                   3744: Автор:
                   3745: Сергей Терентьев (Санкт-Петербург)
                   3746: 
                   3747: Вопрос 6:
                   3748: В Фейсбуке существовало приложение "Социальная ОНА", которое с некоторой
                   3749: вероятностью удаляло профиль пользователя. Назовите ЕЕ одним словом.
                   3750: 
                   3751: Ответ:
                   3752: Рулетка.
                   3753: 
                   3754: Комментарий:
                   3755: Пользователь мог установить приложение, и после запуска с вероятностью
                   3756: один к шести совершалось социальное самоубийство - происходило полное
                   3757: удаление профиля пользователя. Приложение просуществовало всего
                   3758: несколько дней, после чего его заблокировали, ведь оно противоречило
                   3759: политике Фейсбука - социальная сеть должна поощрять увеличение своей
                   3760: аудитории, а не ее сокращение.
                   3761: 
                   3762: Источник:
                   3763: http://www.securitylab.ru/news/440269.php
                   3764: 
                   3765: Автор:
                   3766: Сергей Терентьев (Санкт-Петербург)
                   3767: 
                   3768: Вопрос 7:
                   3769: Во время судебного следствия XIX века выяснилось, что родная деревня
                   3770: подозреваемого Абд аль-Расула была сплошь населена ИМИ и их ремесло
                   3771: передавалось от отца к сыну примерно с XIII века до н.э. Назовите
                   3772: героиню, появившуюся в 1996 году, которая является одной из НИХ.
                   3773: 
                   3774: Ответ:
                   3775: Лара Крофт.
                   3776: 
                   3777: Комментарий:
                   3778: ОНИ - расхитители гробниц. Деревня находится в Египте, который славится
                   3779: гробницами фараонов, а сам Египет - культом мертвых. На протяжении
                   3780: многих веков жители этой деревни находили гробницы и понемногу
                   3781: распродавали найденные в них сокровища. Лара Крофт, расхитительница
                   3782: гробниц, впервые появилась в компьютерной игре 1996 года.
                   3783: 
                   3784: Источник:
                   3785:    1. К. Керам. Боги, гробницы и ученые.
                   3786: http://www.flibusta.is/b/348355/read
                   3787:    2. http://ru.wikipedia.org/wiki/Лара_Крофт
                   3788: 
                   3789: Автор:
                   3790: Сергей Терентьев (Санкт-Петербург)
                   3791: 
                   3792: Вопрос 8:
                   3793: [Ведущему: разборчиво прочитать слово "БубАстис".]
                   3794:    Одной из достопримечательностей города БубАстис является ОНО.
                   3795: Назовите ЕГО двумя словами, начинающимися на одну и ту же букву.
                   3796: 
                   3797: Ответ:
                   3798: Кладбище кошек.
                   3799: 
                   3800: Зачет:
                   3801: Кошачье кладбище.
                   3802: 
                   3803: Комментарий:
                   3804: Название город получил в честь богини-покровительницы кошек - БАстет.
                   3805: Бубастис находится в Египте и в давние времена был центром кошачьего
                   3806: культа, а в самом Египте, как вы помните, был развит культ мертвых.
                   3807: 
                   3808: Источник:
                   3809: http://www.catgallery.ru/kototeka/2012-09-07/bubastis-the-city-of-bastet/
                   3810: 
                   3811: Автор:
                   3812: Сергей Терентьев (Санкт-Петербург)
                   3813: 
                   3814: Вопрос 9:
                   3815: В одной из версий происхождения известного выражения говорится, что,
                   3816: когда кошки не справлялись с мышами, люди надеялись на другого хищника.
                   3817: Тогда-то и нужны были АЛЬФЫ, которые люди надевали и шли в лес на ловлю.
                   3818: Назовите АЛЬФЫ двумя словами.
                   3819: 
                   3820: Ответ:
                   3821: Ежовые рукавицы.
                   3822: 
                   3823: Зачет:
                   3824: Ежиные рукавицы.
                   3825: 
                   3826: Комментарий:
                   3827: Еж - хищник и мышей тоже ест. Но голыми руками поймать его сложно,
                   3828: поэтому изготавливались специальные ежовые рукавицы из толстой кожи.
                   3829: 
                   3830: Источник:
                   3831: http://znaeteli.ru/2011/09/otkuda-vzyalis-ezhovye-rukavicy/
                   3832: 
                   3833: Автор:
                   3834: Сергей Терентьев (Санкт-Петербург)
                   3835: 
                   3836: Вопрос 10:
                   3837: (pic: 20150828.jpg)
                   3838:    Ответьте одним словом, как в Швеции называют подобные транспортные
                   3839: средства.
                   3840: 
                   3841: Ответ:
                   3842: Сквадер.
                   3843: 
                   3844: Комментарий:
                   3845: Чучело "сквадера", про которого было рассказано в одном из предыдущих
                   3846: вопросов, пользуется такой большой популярностью, что слово "сквадер" в
                   3847: современном разговорном шведском языке означает "плохой компромисс" или
                   3848: "сочетание противоречивых элементов". Также этим словом называют гибрид
                   3849: грузовика и автобуса.
                   3850: 
                   3851: Источник:
                   3852: http://ru.wikipedia.org/wiki/Сквадер
                   3853: 
                   3854: Автор:
                   3855: Сергей Терентьев (Санкт-Петербург)
                   3856: 
                   3857: Вопрос 11:
                   3858: Документами Аптекарского приказа XVII века предписывалось тщательно
                   3859: осматривать людей с ИКСОМ. В тех же документах ИКС упоминается под
                   3860: названием "френчь". Назовите ИКС словом, которое начинается и
                   3861: заканчивается на одну и ту же букву.
                   3862: 
                   3863: Ответ:
                   3864: Сифилис.
                   3865: 
                   3866: Комментарий:
                   3867: В ведении Аптекарского приказа находились аптеки, доктора, лекарства.
                   3868: Сифилис появился в России примерно в начале XVI века и назывался по
                   3869: странам, откуда, как считалось, он пришел - польской, немецкой, но чаще
                   3870: французской болезнью. "Френчь" и означает "французская" болезнь. В XVII
                   3871: веке началась планомерная борьба с сифилисом в России. К куртке френч
                   3872: это слово не имеет отношения.
                   3873: 
                   3874: Источник:
                   3875: http://ru.wikipedia.org/wiki/Сифилис
                   3876: 
                   3877: Автор:
                   3878: Сергей Терентьев (Санкт-Петербург)
                   3879: 
                   3880: Вопрос 12:
                   3881: В биографическом фильме Уильям Тёрнер решил посетить одного из ИКСОВ.
                   3882: Наблюдая за работой ИКСА, Уильям спросил, почему тот укрылся, как монах.
                   3883: Назовите ИКСА словом с греческими корнями.
                   3884: 
                   3885: Ответ:
                   3886: Фотограф.
                   3887: 
                   3888: Комментарий:
                   3889: Уильям Тёрнер - известный художник, творивший в первой половине XIX
                   3890: века. Появление фотографии испугало его, и он решил посмотреть на эту
                   3891: новинку. Чтобы снимок не была засвечен, фотограф укрывается идущей от
                   3892: камеры накидкой, что делает его похожим на монаха.
                   3893: 
                   3894: Источник:
                   3895: Х/ф "Уильям Тёрнер" (2014), реж. Майк Ли, 121-я минута.
                   3896: 
                   3897: Автор:
                   3898: Сергей Терентьев (Санкт-Петербург)
                   3899: 
                   3900: Вопрос 13:
                   3901: Говоря об отношении "Журнала общей биологии" к статьям о проблеме
                   3902: происхождения жизни, Кирилл Еськов упомянул ЕГО. Назовите ЕГО двумя
                   3903: русскими или латинскими словами.
                   3904: 
                   3905: Ответ:
                   3906: Вечный двигатель.
                   3907: 
                   3908: Зачет:
                   3909: Perpetuum mobile.
                   3910: 
                   3911: Комментарий:
                   3912: "Журнал общей биологии" Российской Академии наук не рассматривает статьи
                   3913: о проблеме происхождения жизни, как Парижская Академия наук не
                   3914: рассматривает проекты вечного двигателя. Журналы по лингвистике, кстати,
                   3915: не рассматривают статьи на тему происхождения языка.
                   3916: 
                   3917: Источник:
                   3918:    1. К.Ю. Еськов. История Земли и жизни на ней.
                   3919: http://www.flibusta.is/b/329649/read
                   3920:    2. http://naked-science.ru/article/top/nesostoyavshiesya-vechnye-dvig
                   3921: 
                   3922: Автор:
                   3923: Сергей Терентьев (Санкт-Петербург)
                   3924: 
                   3925: Вопрос 14:
                   3926: Гитара - один из символов канадского национального творчества. В честь
                   3927: этого в 2009 году выпустили АЛЬФУ в форме ИКСА. Иногда АЛЬФУ используют
                   3928: вместо ИКСА. Назовите ИКС и АЛЬФУ.
                   3929: 
                   3930: Ответ:
                   3931: Медиатор, монета.
                   3932: 
                   3933: Зачет:
                   3934: Плектр, монета.
                   3935: 
                   3936: Комментарий:
                   3937: Канадский монетный двор отчеканил монету в форме гитарного медиатора.
                   3938: При отсутствии медиатора иногда для игры на гитаре используют монету.
                   3939: 
                   3940: Источник:
                   3941:    1. http://lenta.ru/news/2009/06/04/coin/
                   3942:    2. http://www.mint.ca/store/content/productSeries.jsp?cat=Six%20String%20Nation%20Guitar
                   3943: 
                   3944: Автор:
                   3945: Сергей Терентьев (Санкт-Петербург)
                   3946: 
                   3947: Вопрос 15:
                   3948: В вопросе слово "АЛЬФА" является заменой.
                   3949:    В статье "Вокруг света" об одной из европейских
                   3950: достопримечательностей говорится, что две АЛЬФЫ обещают любовную
                   3951: встречу, три АЛЬФЫ - свадьбу, а четыре АЛЬФЫ, как ни странно, -
                   3952: богатство. Что должна обеспечить одна АЛЬФА?
                   3953: 
                   3954: Ответ:
                   3955: Возвращение [в Рим].
                   3956: 
                   3957: Комментарий:
                   3958: АЛЬФА - монета. Речь идет о римском фонтане Треви. По преданию, две
                   3959: брошенных в фонтан монеты - к любовной встрече, три - к свадьбе, а если
                   3960: вы бросите в фонтан целых четыре монеты, то это сулит, как ни странно,
                   3961: богатство. Одну монету бросают в фонтан, чтобы вернуться в Рим.
                   3962: 
                   3963: Источник:
                   3964: http://www.vokrugsveta.ru/article/203099/
                   3965: 
                   3966: Автор:
                   3967: Сергей Терентьев (Санкт-Петербург)
                   3968: 
                   3969: Вопрос 16:
                   3970: Анри Брейль во время работы в пещере Альтамира использовал мягкие мешки.
                   3971: Забавно, что Альтамиру часто называют "ЕЮ первобытного искусства".
                   3972: Назовите ЕЕ двумя словами.
                   3973: 
                   3974: Ответ:
                   3975: Сикстинская капелла.
                   3976: 
                   3977: Зачет:
                   3978: Сикстинская церковь.
                   3979: 
                   3980: Комментарий:
                   3981: Анри Брейль зарисовывал росписи со свода Альтамиры лёжа, расположившись
                   3982: на мягких мешках. За эти росписи пещеру называют "Сикстинской капеллой
                   3983: первобытного искусства". По легенде, Микеланджело расписывал потолок
                   3984: Сикстинской капеллы лёжа. Кстати, этот вопрос - шестнадцатый. :-)
                   3985: 
                   3986: Источник:
                   3987:    1. http://tripmydream.com/ru/blog/neobychnye-mesta/peschera-altamira
                   3988:    2. http://ru.wikipedia.org/wiki/Потолок_Сикстинской_капеллы
                   3989:    3. Е.Г. Дэвлет. Альтамира у истоков искусства. - М.: Алетейа, 2004. -
                   3990: С. 23.
                   3991: 
                   3992: Автор:
                   3993: Сергей Терентьев (Санкт-Петербург)
                   3994: 
                   3995: Вопрос 17:
                   3996: МамврИйский дуб также называют дубом Авраама. До полного засыхания
                   3997: дерева считалось, что засохшая часть дуба символизирует ТАКОЙ ИКС, а
                   3998: зеленая - СЯКОЙ ИКС. Назовите ТАКОЙ ИКС и СЯКОЙ ИКС.
                   3999: 
                   4000: Ответ:
                   4001: Ветхий завет, Новый завет.
                   4002: 
                   4003: Зачет:
                   4004: В любом порядке.
                   4005: 
                   4006: Комментарий:
                   4007: Под Мамврийским дубом, согласно Библии, Авраам принял Бога. Дуб
                   4008: пользуется популярностью у паломников, которые дали частям дерева
                   4009: подобное толкование. Впрочем, зеленой части дуба уже не существует -
                   4010: благодаря паломникам, отрывавшим от дерева куски коры как реликвию.
                   4011: 
                   4012: Источник:
                   4013: http://ru.wikipedia.org/wiki/Мамврийский_дуб
                   4014: 
                   4015: Автор:
                   4016: Сергей Терентьев (Санкт-Петербург)
                   4017: 
                   4018: Вопрос 18:
                   4019:    <раздатка>
                   4020:    ... вот, человек стал как один из нас...
                   4021:    ... сойдем же и смешаем там язык их...
                   4022:    </раздатка>
                   4023:    Айзек АзИмов пишет, что создатели Библии так и не смогли избавиться
                   4024: от НЕГО. Назовите ЕГО словом с двумя корнями.
                   4025: 
                   4026: Ответ:
                   4027: Политеизм.
                   4028: 
                   4029: Зачет:
                   4030: Многобожие; многобожество.
                   4031: 
                   4032: Комментарий:
                   4033: Несмотря на то что в Библии один Бог, в ней, по мнению Азимова, можно
                   4034: увидеть намек на более ранние политеистические религии.
                   4035: 
                   4036: Источник:
                   4037: А. Азимов. Путеводитель по Библии. http://www.flibusta.is/b/353503/read
                   4038: 
                   4039: Автор:
                   4040: Сергей Терентьев (Санкт-Петербург)
                   4041: 
                   4042: Вопрос 19:
                   4043: [Ведущему: читать четверостишие медленно, чтобы команды успели его
                   4044: записать.]
                   4045:    Восстановите пропущенную строку в эпиграмме на фильм о крушении
                   4046: поезда с цирковым вагоном:
                   4047:    "Фильм катастрофами не скуден:
                   4048:    Пожар - вовсю. Сюжет - "убойный".
                   4049:    СТРОКА ПРОПУЩЕНА
                   4050:    И только зрители спокойны!".
                   4051: 
                   4052: Ответ:
                   4053: Смешались в кучу кони, люди.
                   4054: 
                   4055: Комментарий:
                   4056: Речь идет о фильме "34-й скорый". Помимо людей в поезде, в цирковом
                   4057: вагоне перевозили лошадей.
                   4058: 
                   4059: Источник:
                   4060: http://ru.wikipedia.org/wiki/34-й_скорый
                   4061: 
                   4062: Автор:
                   4063: Сергей Терентьев (Санкт-Петербург)
                   4064: 
                   4065: Вопрос 20:
                   4066: [Ведущему: разборчиво прочитать слово "ОрхОну".]
                   4067:    Император ЮнчжЭн за помощь в победе над неприятелем пожаловал ОрхОну
                   4068: княжеский титул и выделил триста лян серебра ежегодного содержания.
                   4069: Какой правитель, по мнению автора вопроса, поступил однажды
                   4070: противоположным образом?
                   4071: 
                   4072: Ответ:
                   4073: Ксеркс I.
                   4074: 
                   4075: Зачет:
                   4076: Ксеркс.
                   4077: 
                   4078: Комментарий:
                   4079: Орхон - река в Монголии. Существует легенда, что, когда враги императора
                   4080: побежали в сторону Орхона, они все утонули в нем. В качестве
                   4081: благодарности император приказал ежегодно сбрасывать серебро в воду
                   4082: реки. Если ЮнчжЭн воздал водному объекту почести, то Ксеркс приказал
                   4083: высечь море.
                   4084: 
                   4085: Источник:
                   4086:    1. http://necrotula.ru/monastyr/erdeni-dzu
                   4087:    2. http://ru.wikipedia.org/wiki/Орхон
                   4088:    3. http://ru.wikipedia.org/wiki/Ксеркс_I
                   4089: 
                   4090: Автор:
                   4091: Сергей Терентьев (Санкт-Петербург)
                   4092: 
                   4093: Вопрос 21:
                   4094: Пауков семейства filistatidae [филистатИдэ] за умение проползать сквозь
                   4095: узкие вертикальные дыры называют пауками-ИКСАМИ. В Финляндии могут
                   4096: наложить штраф, если домовладелец не обращается к услугам ИКСА. Назовите
                   4097: ИКСА.
                   4098: 
                   4099: Ответ:
                   4100: Трубочист.
                   4101: 
                   4102: Комментарий:
                   4103: Трубочисты тоже по своей профессии пролезают сквозь узкие вертикальные
                   4104: трубы. В финских домах печь или камин - не редкость. Если финны,
                   4105: владельцы таких домов, не производят чистку дымохода, то их ждет штраф,
                   4106: так как засоренный дымоход может стать причиной пожара.
                   4107: 
                   4108: Источник:
                   4109:    1. http://ru.wikipedia.org/wiki/Filistatidae
                   4110:    2. http://www.stopinfin.ru/archive/171/16268/
                   4111: 
                   4112: Автор:
                   4113: Сергей Терентьев (Санкт-Петербург)
                   4114: 
                   4115: Вопрос 22:
                   4116: Жан-Франсуа Шампольон озаглавил одну из своих работ "Хронология от Адама
                   4117: до Шампольона-младшего". Тем самым он пытался сгладить поступок старшего
                   4118: брата, который отказался от НЕЕ, рано поняв, что Жан-Франсуа станет
                   4119: всемирно известным человеком. Напишите ЕЕ.
                   4120: 
                   4121: Ответ:
                   4122: Шампольон.
                   4123: 
                   4124: Зачет:
                   4125: Фамилия Шампольон; фамилия.
                   4126: 
                   4127: Комментарий:
                   4128: Старший брат понял, что Жан-Франсуа станет известным человеком и
                   4129: прославит фамилию, поэтому предпочитал оставаться в тени. Он добавил к
                   4130: своей фамилии город рождения - ФижАк, а потом и вовсе попросил называть
                   4131: его просто "Фижак", без фамилии. Названием своей работы Жан-Франсуа
                   4132: хотел подчеркнуть, что у него есть старший брат по фамилии Шампольон.
                   4133: 
                   4134: Источник:
                   4135: К. Керам. Боги, гробницы и ученые. http://www.flibusta.is/b/348355/read
                   4136: 
                   4137: Автор:
                   4138: Сергей Терентьев (Санкт-Петербург)
                   4139: 
                   4140: Вопрос 23:
                   4141: БельдЮга - мелкая рыба. По одной из версий, в известном произведении
                   4142: вместо НИХ изначально говорилось о рыбаках, которые ловили бельдюгу.
                   4143: Назовите ИХ.
                   4144: 
                   4145: Ответ:
                   4146: Биндюжники.
                   4147: 
                   4148: Комментарий:
                   4149: По одной из версий, в песне "Шаланды, полные кефали" изначально пелось
                   4150: не "все биндюжники вставали", а "все бельдюжники вставали", выказывая
                   4151: тем самым уважение настоящему рыбаку - Косте, который приезжал с
                   4152: кефалью, более крупной рыбой.
                   4153: 
                   4154: Источник:
                   4155:    1. http://www.bolshoyvopros.ru/questions/618074-chem-na-rabote-zanimalis-odesskie-bindjuzhniki-vstavavshie-pri-vide-kosti.html
                   4156:    2. http://ru.wikipedia.org/wiki/Шаланды,_полные_кефали
                   4157: 
                   4158: Автор:
                   4159: Сергей Терентьев (Санкт-Петербург)
                   4160: 
                   4161: Вопрос 24:
                   4162: В 1969 году журналисты газеты "Правда" СтрЕльников и ШатунОвский
                   4163: повторили маршрут и написали книгу "ОНА справа и слева". Назовите ЕЕ
                   4164: одним словом.
                   4165: 
                   4166: Ответ:
                   4167: Америка.
                   4168: 
                   4169: Комментарий:
                   4170: Журналисты повторили маршрут Ильфа и Петрова, чтобы посмотреть, как
                   4171: изменилась Америка за треть века. Путешествие тоже было автомобильным, и
                   4172: журналисты описывали то, что видели из окон машины.
                   4173: 
                   4174: Источник:
                   4175: http://ru.wikipedia.org/wiki/Одноэтажная_Америка
                   4176: 
                   4177: Автор:
                   4178: Сергей Терентьев (Санкт-Петербург)
                   4179: 
                   4180: Вопрос 25:
                   4181: ПОднятые ворота на гербе Братиславы символизируют ЭТО, присущее жителям
                   4182: города. Назовите ЭТО словом с двумя корнями.
                   4183: 
                   4184: Ответ:
                   4185: Гостеприимство.
                   4186: 
                   4187: Зачет:
                   4188: Хлебосольство.
                   4189: 
                   4190: Комментарий:
                   4191: Поднятые ворота будто показывают, что гостям будут рады.
                   4192: 
                   4193: Источник:
                   4194: http://ru.wikipedia.org/wiki/Герб_Братиславы
                   4195: 
                   4196: Автор:
                   4197: Сергей Терентьев (Санкт-Петербург)
                   4198: 
                   4199: Вопрос 26:
                   4200: По выражению одного из создателей, ИКС - это всё, что находится ниже
                   4201: четырнадцатой улицы. По одной из версий, название ИКСА означает
                   4202: "проклятый Богом". Назовите ИКС.
                   4203: 
                   4204: Ответ:
                   4205: ГОтэм-сити.
                   4206: 
                   4207: Зачет:
                   4208: ГОтэм.
                   4209: 
                   4210: Комментарий:
                   4211: Один из первых авторов Бэтмена объяснял, что Метрополис - это Нью-Йорк
                   4212: выше 14-й улицы, а Готэм - ниже 14-й улицы. Неблагополучная история
                   4213: Готэма связывается с неудачно выбранным названием, в котором можно
                   4214: увидеть искаженную фразу "goddamn" [годдэмн] - "проклятый Богом".
                   4215: 
                   4216: Источник:
                   4217: http://ru.wikipedia.org/wiki/Готэм-сити
                   4218: 
                   4219: Автор:
                   4220: Сергей Терентьев (Санкт-Петербург)
                   4221: 
                   4222: Вопрос 27:
                   4223: [Ведущему: разборчиво прочитать слово "беотУки".]
                   4224:    По одной из версий, ЭТО ПРОЗВИЩЕ появилось из-за того, что беотУки
                   4225: пользовались охрой. Недавно американский суд постановил сменить название
                   4226: спортивного клуба, поскольку оно содержало ЭТО ПРОЗВИЩЕ. Напишите ЭТО
                   4227: ПРОЗВИЩЕ из одного слова.
                   4228: 
                   4229: Ответ:
                   4230: Краснокожие.
                   4231: 
                   4232: Зачет:
                   4233: Redskins.
                   4234: 
                   4235: Комментарий:
                   4236: Беотуки - обитатели острова Ньюфаундленд, вымершие в XIX веке. Они были
                   4237: одним из первых индейских племен, вступившим в контакт с европейцами. По
                   4238: одной из версий, за то, что беотуки раскрашивали одежду и лицо охрой,
                   4239: они и получили прозвище "краснокожие". По мнению суда США, название
                   4240: "Washington Redskins" [вашингтон редскинз], т.е. "Вашингстонские
                   4241: краснокожие", оскорбляет коренное население страны.
                   4242: 
                   4243: Источник:
                   4244:    1. http://ru.wikipedia.org/wiki/Беотуки
                   4245:    2. http://www.eurosport.ru/american-football/nfl/2015-2016/story_sto4813926.shtml
                   4246: 
                   4247: Автор:
                   4248: Сергей Терентьев (Санкт-Петербург)
                   4249: 
                   4250: Вопрос 28:
                   4251: Модельер из произведения Брюса ЧАтвина вспоминает, что кубинки были ее
                   4252: постоянными покупательницами до тех пор, пока в Европе не начали активно
                   4253: выращивать ЕЕ. Назовите ЕЕ двумя словами, начинающимися на одну и ту же
                   4254: букву.
                   4255: 
                   4256: Ответ:
                   4257: Сахарная свекла.
                   4258: 
                   4259: Комментарий:
                   4260: В Европе начали добывать сахар из свеклы. Кубинский тростниковый сахар
                   4261: упал в цене, и мужья кубинок уже не зарабатывали таких больших денег.
                   4262: 
                   4263: Источник:
                   4264: Б. Чатвин. "Утц" и другие истории из мира искусств.
                   4265: http://www.flibusta.is/b/378066/read
                   4266: 
                   4267: Автор:
                   4268: Сергей Терентьев (Санкт-Петербург)
                   4269: 
                   4270: Вопрос 29:
                   4271: Кубинский правитель Фульхенсио Батиста способствовал появлению в Гаване
                   4272: азартных заведений. В статье Википедии Гавана названа ИМ. Назовите ЕГО
                   4273: двумя словами, начинающимися на одну и ту же букву, первое из которых
                   4274: является двухкоренным.
                   4275: 
                   4276: Ответ:
                   4277: Латиноамериканский Лас-Вегас.
                   4278: 
                   4279: Комментарий:
                   4280: Куба относится к Латинской Америке. В США азартными играми славится
                   4281: город Лас-Вегас.
                   4282: 
                   4283: Источник:
                   4284:    1. http://ru.wikipedia.org/wiki/Батиста,_Фульхенсио
                   4285:    2. http://ru.wikipedia.org/wiki/Латинская_Америка
                   4286: 
                   4287: Автор:
                   4288: Сергей Терентьев (Санкт-Петербург)
                   4289: 
                   4290: Вопрос 30:
                   4291: В фильме Кубрика "Доктор Стрейнджлав" посреди "Военной комнаты" стоит
                   4292: большой стол для совещаний, за которым разыгрывают судьбы мира. По
                   4293: замыслу Кубрика, для стола должно было использоваться ОНО, но фильм стал
                   4294: черно-белым и идею бы не оценили. Назовите ЕГО двумя словами,
                   4295: начинающимися на парные согласные.
                   4296: 
                   4297: Ответ:
                   4298: Зеленое сукно.
                   4299: 
                   4300: Комментарий:
                   4301: Большой стол с зеленым сукном напоминал бы стол для покера, за которым
                   4302: главы государств разыгрывают судьбу мира.
                   4303: 
                   4304: Источник:
                   4305: http://horror-movies.ru/Reviews-1/Dr_Strangelove_or_How_I_Learned_to_Stop_Worrying_and_Love_Bomb.htm
                   4306: 
                   4307: Автор:
                   4308: Сергей Терентьев (Санкт-Петербург)
                   4309: 
                   4310: Вопрос 31:
                   4311:    <раздатка>
                   4312:    weed - марихуана
                   4313:    </раздатка>
                   4314:    В 1976 году штат Калифорния частично декриминализовал марихуану. На
                   4315: радостях кто-то ночью видоизменил ЭТО. Назовите ЭТО.
                   4316: 
                   4317: Ответ:
                   4318: Знак "Hollywood".
                   4319: 
                   4320: Зачет:
                   4321: По словам "Hollywood" или "Голливуд".
                   4322: 
                   4323: Комментарий:
                   4324: Кто-то изменил знак "Hollywood", который находится в Лос-Анджелесе,
                   4325: штате Калифорния, заменив буквы "o" на "e".
                   4326: 
                   4327: Источник:
                   4328:    1. http://mi3ch.livejournal.com/3090985.html
                   4329:    2. http://en.wikipedia.org/wiki/Hollywood_Sign
                   4330: 
                   4331: Автор:
                   4332: Сергей Терентьев (Санкт-Петербург)
                   4333: 
                   4334: Вопрос 32:
                   4335: Нефтяной корпорации "Марафон" принадлежит симметричное ЭТО. Напишите ЭТО
                   4336: корпорации "Марафон", состоящее из семи символов.
                   4337: 
                   4338: Ответ:
                   4339: moc.com.
                   4340: 
                   4341: Комментарий:
                   4342: На английском компания называется "Marathon Oil Corporation" [марафон
                   4343: ойл корпорэйшн], сокращенно - MOC. Компании принадлежит уникальное
                   4344: симметричное доменное имя - moc.com.
                   4345: 
                   4346: Источник:
                   4347: http://moc.com
                   4348: 
                   4349: Автор:
                   4350: Сергей Терентьев (Санкт-Петербург)
                   4351: 
                   4352: Вопрос 33:
                   4353: Авторы одной книги пишут, что эффект от НЕГО быстро сошел на нет и,
                   4354: например, при виде волн дамы уже не подбирали юбки. Назовите ЕГО двумя
                   4355: словами, начинающимися на одну и ту же букву.
                   4356: 
                   4357: Ответ:
                   4358: Прибытие поезда.
                   4359: 
                   4360: Зачет:
                   4361: Приезд поезда.
                   4362: 
                   4363: Комментарий:
                   4364: В 1900 году кинематограф уже не поражал воображение людей, сделавшись
                   4365: обыденностью. А ведь незадолго до этого люди не на шутку испугались
                   4366: поезда на экране.
                   4367: 
                   4368: Источник:
                   4369: Ю.М. Лотман, Ю.Г. Цивьян. Диалог с экраном. - Таллин: Александра, 1994.
                   4370: 
                   4371: Автор:
                   4372: Сергей Терентьев (Санкт-Петербург)
                   4373: 
                   4374: Вопрос 34:
                   4375: Бен ТЕрпин, комедийный актер немого кино, боялся, что ЭТО может
                   4376: пропасть, например, от удара по голове, и застраховал ЭТО на большую
                   4377: сумму. Назовите ЭТО словом с двумя корнями.
                   4378: 
                   4379: Ответ:
                   4380: Косоглазие.
                   4381: 
                   4382: Зачет:
                   4383: Гетеротропия.
                   4384: 
                   4385: Комментарий:
                   4386: Зрители покатывались с хохоту при виде лица Бена с глядящими в разные
                   4387: стороны глазами. Косоглазие он заработал после несчастного случая и
                   4388: после каждого удара по голове бежал к зеркалу, чтобы проверить, не
                   4389: прошло ли косоглазие. Бен относился к тем немногим людей с косоглазием,
                   4390: которые не пытались избавиться от недостатка, а были готовы заплатить
                   4391: большие деньги, чтобы оно сохранилось.
                   4392: 
                   4393: Источник:
                   4394: http://expert.ru/2013/03/25/zolotoj-yazyik/
                   4395: 
                   4396: Автор:
                   4397: Сергей Терентьев (Санкт-Петербург)
                   4398: 
                   4399: Вопрос 35:
                   4400: ЕМУ принадлежит изобретение под названием "мерситрОн", представляющее
                   4401: собой штатив с тремя капельницами. Напишите ЕГО фамилию.
                   4402: 
                   4403: Ответ:
                   4404: Кеворкян.
                   4405: 
                   4406: Комментарий:
                   4407: Название изобретения происходит от слова "mercy" [мёрси] -
                   4408: "сострадание". В одной из капельниц содержится сильное снотворное, в
                   4409: другой - вещество, парализующее мышцы, в третьей - хлорид калия,
                   4410: останавливающий сердце. Джек Кеворкян известен как популяризатор
                   4411: эвтаназии.
                   4412: 
                   4413: Источник:
                   4414:    1. http://www.kommersant.ru/doc/2286622
                   4415:    2. http://ru.wikipedia.org/wiki/Кеворкян,_Джек
                   4416: 
                   4417: Автор:
                   4418: Сергей Терентьев (Санкт-Петербург)
                   4419: 
                   4420: Вопрос 36:
                   4421: Леонид Парфёнов в фильме о Крымской войне говорит, что сообщение о
1.3       rubashki 4422: приближении врага достигло города во время театральной постановки, и
1.1       rubashki 4423: упоминает прилагательное с двумя приставками. Напишите это
                   4424: прилагательное.
                   4425: 
                   4426: Ответ:
                   4427: Пренеприятнейшее.
                   4428: 
                   4429: Комментарий:
                   4430: В фильме говорится, что постановку "Ревизора" пришлось отменить из-за
                   4431: получения пренеприятнейшего известия.
                   4432: 
                   4433: Источник:
                   4434: http://www.youtube.com/watch?v=eGVhsV2hHl8&t=25m10s
                   4435: 
                   4436: Автор:
                   4437: Сергей Терентьев (Санкт-Петербург)
                   4438: 

FreeBSD-CVSweb <freebsd-cvsweb@FreeBSD.org>